PDA

View Full Version : Monks: How do they feel?



Sindal
2018-11-23, 11:40 PM
Hi yall

So I've never played a monk. Certainly wouldn't mind try it (acutally considered it as my first melee class since my two other chracters thus far are somewhat ranged) but until then...
Looking at their skill lines and perks, they seem pretty self sufficient and functional. But that's usually where it stops. Most people, if i glance, will say that monks are 'ok'. So that doesn't tell me if people are saying they're happy or they wish there was more.

So to the people who play them: Whats it like playing a monk.
Is there anything you would change? Do you feel good playing it? Are there things you wish you had or things you wish got a little bump?

I'm reminded when people complain about the ranger (that they're functional but are a bit of a mess) and sorcerors (who are functional but require some of spell planning to make work properly)
Is this the same kinda issue or am making an assumption

Nifft
2018-11-23, 11:50 PM
Open Hand and Shadow are both great (from what I've seen).

Haven't seen anyone play a Four Elements variant, nor any of the others.

In specific, Open Hand is great at combat, and Shadow is great at mobility (level 6+).

Deathtongue
2018-11-23, 11:52 PM
Hi yall

So I've never played a monk.
Looking at their skill lines and perks, they seem pretty self sufficient and functional. But that's usually where it stops. Most people, if i glance, will say that monks are 'ok'. So that doesn't tell me if people are saying they're happy or they wish there was more.

So to the people who play them: Whats it like playing a monk.
Is there anything you would change? Do you feel good playing it? Are there things you wish you had or things you wish got a little bump?

I'm reminded when people complain about the ranger (that they're functional but are a bit of a mess) and sorcerors (who are functional but require some of spell planning to make work properly)
Is this the same kinda issue or am making an assumptionHonestly? You're kind of right. Maybe it's just because the monks I've seen don't use Stunning Fist enough or use their ki points wisely enough, but in my experience don't have a 'wow' class feature like a Fighter throwing down with Action Surge + 6 GMW attacks or a Paladin burning a 4th-level spell slot on a crit or a wizard breaking out Wall of Force or just a Bear Totem Barbarian evenly trading blows with an Adult Black Dragon.

And I think it's because the monk class design was formulated before game designers realized that both feats, multiclassing, and magic items were here to stay.

Monks are relatively better compared to other classes in games that don't use feats, because with the possible exception of Magic Initiate: Warlock (for Hex) there aren't really many feats that benefit monks. Old standbys like Medium Armor Proficiency or Shield Master or Sharpshooter or Resilient are game-changers for a lot of classes in a way they aren't for monks.

Going with the above, Monks also don't benefit as much from magical items as other classes. A good magical sword or magical armor or staff is a total gamechanger for paladins and barbarians and warlocks. Yes, monks really benefit from Belts of Giant Strength if they can get them to drop and the Claw Insignia helps, but aside from that there really aren't any items that supercharge a monk's effectiveness the way, say, an Oathbow makes a Fighter a DPR hardcase.

The above two things is why questionable early-edition classes like the Ranger seem to benefit more from expansion material than the Monk.

I think monks need a couple of killer app feats, like something that lets them get -5/+10 for finesse weapons/unarmed strikes and a few monk-only magical items.

Astofel
2018-11-24, 12:08 AM
Personally I love monks, they're one of my favourite classes in 5e. The subclasses I've played have all been great and created a distinct play style, but most of all it just feels cool to do ridiculous monk things, like being way faster than any other player and running on water. Take Mobile and/or be a tabaxi and marvel at your ability to be literally anywhere on the battlefield.

Grimmnist
2018-11-24, 12:24 AM
I have not played a monk, however I have played with one and DM'ed two so I feel comfortable giving an overview. Compared to Rangers or Sorcerers Monks have a higher floor but a lower ceiling. Sorcerers in the right situation are devastating in combat and Rangers are the ultimate wilderness skill monkey (assuming you're in the right terrain populated by the right enemies). Monk on the other hand never really shines, this isn't to say it's bad, but rather very consistent.

Mobility - This is probably my favorite feature of the monk, they are quite mobile gaining extra movement speed and have some really cool flavor such as running up walls, what's more they don't have to spend major resources to use their special movement. However, casters with Misty Step or Dimension Door can obviously move better than the monk. Since most monks don't have ranged attacks -Edit: since monks do less damage while at range-, the movement, despite being good, can at times be lacking.

Offence - The monk is a pretty good, consistent, offensive character. The monk gets a lot of attacks by using Flurry of Blows so you're very unlikely to come out of a turn without doing any damage, but the damage never spikes as high as other classes.

Defense - The monk is really good at defense, their AC is between 15-20 depending on level which is comparable to other Martial classes. However, there AC is based on Dex + Wis so you don't get the experience of upgrading your armor if that matters to you. They can catch arrows out of the air which is a really cool flavor and works pretty well. They can also use a dodge bonus action by spending a ki point, and with the Evasion feature the character is very hard to hurt.

Skills - The two primary stats of the Monk are Dex and Wis which are both great skill abilities, however the monk doesn't get extra profs so again, just pretty good.

I would recommend choosing to play Monk after knowing what everyone else is playing. Monk can fill a number of roles pretty well so it is a great martial gap filler for the party. However, in the wrong party you will still be useful but never the star of the show.

-Edit: I should also point out that they have a unique flavor so that can absolutely be a fun roleplay-

stoutstien
2018-11-24, 12:32 AM
Well the weird thing about monks is that their class role changes as the reach different tiers of play
1-5 mobile striker
5-7 control via stunning strike
7-10 back to mobile strike but now cc and lots of more survivability.
10-13 social butterfly with auto languages(this a joke but it does suddenly make them quite useful if use a lot of npc with different languages)
14+ can actually tank quite well. At this point you have the ki to ba Dodge and Prof in all saving throws can go a long way.
Then at lv 18 you basically have reist on top of that.
Stunning strike really is the crowning jewel due to stealing action economy and chewing up legendary saves. Low cost high reward.

Zalabim
2018-11-24, 04:23 AM
Since most monks don't have ranged attacks, the movement, despite being good, can at times be lacking.

This bothers me to no end. Monks are proficient in simple weapons, focus on dexterity, and get extra attack. They're not as dangerous with ranged attacks as they are with melee, but with the defenses you mention (deflect missiles, evasion, patient defense) they're also in a lot less danger.

Even if you don't want to carry a shortbow, every monk should at least have a humble sling.

Appleheart
2018-11-24, 05:45 AM
I like the monk, a lot. I do agree though that the class is, in its pure form, on the weaker end of things.

A great solution, I have found at least, is to run a Kensai monk, and multi-class in a bit of Beast Master ranger (we run Revised Ranger at my table, which helps a lot with this).

Dueling fighting style, favored enemy, Hunter's Mark, and a nice pet companion are great compliments to the monk's base kit that helps ramp up the damage, etc. The kensai also gets a decent boost to the ranged capabilities of the class. The 3 or 5 levels that I invested into Ranger felt really good.

Wood Elf and Ghostwise Halflings are both already the perfect races for monks, statwise, and both fit well with the theme and flavor of adding a splash of ranger. It lets you really play like a self-sufficient master of the wilds. :)

Sahe
2018-11-24, 05:49 AM
I really like Monks, sadly I haven't played them much myself.

In a game I DM I let my players roll for stats and the Variant Human Monk Player ended up with a 20 in dex after everything was said and done...and let's just say...she is absolutely scary.

She has an AC of 18, 2 to 3 Attacks each round which each deal 6 Minimum Damage and her consistency, basically makes her Batman (she also doesn't kill humanoids and animals).

She went off solo at level 2 and was ambushed by a Nothic, a CR2 creature...she ripped it apart. The amount of damage she deals is unreal and she is responsible for at least half the party's takedowns all on her own (other party members are a cleric, barbarian and druid...and the barbarian even has a +1 sword.)

Her character is a bit lacking in social situations, but that it's also due to the fact that the character's voice was stolen by the BBEG.

LordNibbler
2018-11-24, 07:38 AM
I have a shadow monk who is great at disrupting the enemy. He gets to places the DM doesn’t want him. He is stunning the caster hiding behind rows of melee fighters. He is running across rivers and up walls. He is dropping darkness in inconvenient places. As a wood elf with the Mobile feat (let’s you spend your bonus action on extra attacks or shadow step instead of disengaging which the feat gives for free), the high move speed is almost like having a ranged attack!
In short, I have a great time playing him though I have to be somewhat tactical and pay attention to action economy and ki management.

Citan
2018-11-24, 08:18 AM
Hi yall

So I've never played a monk. Certainly wouldn't mind try it (acutally considered it as my first melee class since my two other chracters thus far are somewhat ranged) but until then...
Looking at their skill lines and perks, they seem pretty self sufficient and functional. But that's usually where it stops. Most people, if i glance, will say that monks are 'ok'. So that doesn't tell me if people are saying they're happy or they wish there was more.

So to the people who play them: Whats it like playing a monk.
Is there anything you would change? Do you feel good playing it? Are there things you wish you had or things you wish got a little bump?

I'm reminded when people complain about the ranger (that they're functional but are a bit of a mess) and sorcerors (who are functional but require some of spell planning to make work properly)
Is this the same kinda issue or am making an assumption
Hi!

Well, I'd say "both" actually. XD
The thing is, whereas Fighters/Rogues/Barbarians (and to a lesser extent Paladins and Rangers) play roughly the same from level 1 to level 20, Monk's playstyle is much more fluctuant in style as you level up.

The core word of Monk though is: "tactical". More than Fighter and Rangers, *much* more than Barbarians or even Paladins, and even more than Rogue.
Reason is that most of their skills (outside archetypes) is melee focused, with mobility-based resilience for half their life, while "non-linearly" becoming the ultimate tank in the second half due to top features and more ASIs.

---
So when you start a Monk, you'll first fight by carefully choosing one or two enemies max at a time, using thrown/ranged weapons as often as possible, and use most Ki on Dodge/Dash/Disengage.
When you get your first archetype feature, you'll sometimes use Ki on that feature especially when a rest is expected next (or if you're out of combat for Shadow/4e).

---
Although you get Extra Attack and Stunning Strike at level 5, it won't revolution your playstyle... Yet.
Unless of course you picked Mobile (meaning you can blow Ki on Flurry more easily since you won't fear OAs) or Elven Accuracy (meaning the sooner you land a Stunning Strike the sooner you -and others- will get high nova damage) instead of plain DEX bump.
Otherwise, you'll overall still play the same: focus on evading enemies, with occasional ki(s) spent on base Monk features or archetype features because you *really* need it.

After that, from level 6 to level 10, you're still playing mostly the same except that higher level means higher ki pool meaning the number of times you can do awesome things increase.
That's precisely the reason why Open Hand and Shadow are so liked by people: their abilities cost little overall considering the expected benefit.

---
Note could be said about level 9 though: ability to run on wall and liquids seems anecdotical, but it is actually a game-changer in many closed-areas fightsBecause zones that enemies thought "safe from enemies" thanks to a frontline blockade are actually very reachable by you (walls running, even ceiling could be used -small houserule but for short distances like 5-10 feet I'd say most DM would be fine with it). Furthermore, by that time, you got 15-20 bonus feet speed, + base speed (lets say 30 because most common) so with a Ki on Dash you get an effective "45-50 way&back" speed.

Same could be said for places fortified through environmental engineering (read: fortress with large water zone around for example). You are the only one who can, in one or two turns max, go from "pinned downed by archers across the water plan" to "pummeling said archers on top of the wall".

The first big boost really comes from levels 11 to 15 though.
Depending on archetypes, you get "bland but effective" or "very great" features, but the more important is that you get even more Ki and mobility and proficiency in all saves (+ half-advantage for a ki), ability to speak all languages (actually much more useful than one would expect unless DM waived all such considerations in the first place) and another ASI to boost resilience.
You can now proudly stand on the frontline when facing casters and archers, and you can easily stand in the middle of attacks because you can spare a Dodge as bonus action when necessary. You are still way behind any Barbarian or some Fighters when party needs a sturdy meat shield though (read: party wants one to actually take hits ^^).
Thanks to "all languages", you can now be the best man of the party in foreign countries, either as an intermediate for negotiations for example or by directly taking spying/negociating in your own hands.

Back on archetypes and my dichotomy: Kensei, Long Death and Drunken gets "bland but effective" (read: won't change anything to how you play or things you can do). Shadow and Sun Soul roughly in the same patch (I expect a Shadow to be sneaking/spying, 11th just makes it much easier. I expect a Sun Soul to do some blasting, lvl 11 just makes it easier).
Open Hand's level 11 feature is actually interesting for an out-of-combat use, like going into heated negociations (with risk of rupture breaking into physical conflict), sneaking/adventuring in places when you expect an ambush is possible, and other things like that. Wouldn't say it's "powerful" though. Just interesting. ^^
4E is the oddball: depending on your choices up to that point, and the choices at that point, lvl 11 may be a simple -but big- upgrade to being a Monk (high speed -> fly), a simple -but big- upgrade to being a blaster (Shatter -> Fireball), or a new different rope by picking one of the more distinctive abilities (sneaking? Gaseous Form. Forced movement? WaterWhip/UnbrokenAir).

---
And on the last quarter, it's sheer awesomeness level by level.
Not only do all Monks get Empty Body and Perfect Self, some archetypes get powerful features (Open Hand is with little doubt the best, next come 4E then Shadow then others).
By that time, you are simply the best or second best martial to get in party in a vast array of situations, in competition with Paladin.

How many get there though? That is the main problem. :)

----- TL DR ----
In short, Monk is a class that evolves more than others with levels up, but stays, in essence, a class that require fine reading of tactical situation and fair assessment of threats and self-potency because all its power could be summed up (of course exagerating a bit here) as "woosh around the battlefield like an angry, poisonous stinging bee".
If you don't like to play as a brainy martial, it's not the best fit. ^^

If you want to play a Monk, I'd recommend Open Hand for starters simply because it's the Monkishest of all. If you like tactical play, 4E and Shadow would be my recommandations.
In any case, I'd suggest you try one when you get a chance to play in a party that can reliably get Rope Trick or Catnap, and at least Leomund's Tiny Hut (alternative in some campaigns: Water Breathing). That way you don't have to worry too much about Ki consumption. :)

sophontteks
2018-11-24, 08:36 AM
Monks feel like ninjas. They can scale walls and walk on water at their movement speed, which is very fast. If the party was confronted by archers firing over a castle wall the monk could run over the mote, up the wall, and stun the commander all while catching the arrows being fired at him. Their ability to circumvent barriers and break through lines into the squishy heart of the enemy is what sets them apart.

They are the bane of all ranged users, magic or not. No amount of cover or bodyguards will stop a monk from reaching them and taking them out of the fight completely with a stunning fist. Because monks fire off many attacks each round its good bye concentration. Defensively monks excel against both spells and arrows as well with immunities, bonuses to all their saves, and the ability to catch arrows that hit them.

Of all the archtypes I like Drunken Monk the best. They don't sound amazing on paper, but that free disengage plus 10 feet of movement at level 3 is just freaking incredible. Most monks have to sacrifice an attack to run past melee combatants. Drunken monks keep their flurry and get a hefty 10 feet movement speed on top of it to help them reach that priority target without needing to dash.

Of all the races, I like the halflings here. Many attacks plays into the halfling luck really nice, the enemy can't block the path vs. a halfling, and halflings can hide behind other people. So the enemy moves in against the mage. Little do they know, there is a 30 pound ball of fisted fury hiding under their skirt.

Nifft
2018-11-24, 11:38 AM
Of all the races, I like the halflings here. Many attacks plays into the halfling luck really nice, the enemy can't block the path vs. a halfling, and halflings can hide behind other people. So the enemy moves in against the mage. Little do they know, there is a 30 pound ball of fisted fury hiding under their skirt.

Codshatter, the sexy shoeless god of war and bane of codpieces everywhere?

sophontteks
2018-11-24, 11:40 AM
Codshatter, the sexy shoeless god of war and bane of codpieces everywhere?
This is the best thing I heard all day and I don't even think I fully grasp the reference.

SociopathFriend
2018-11-24, 12:13 PM
Admittedly my last run-through as a monk was short-lived (I don't remember why) but from what I've seen of others using the class- you have to really adapt to your chosen style. They're not like Fighters/Barbarians/Paladins where walking up to the enemy and unloading on them is always the go-to move and they're not like Rangers/Rogues where hanging back and peppering with ranged attacks is the thing you always do.

Monks ride the line in-between. They're meant to go in, lay a smack down, and then get out. As such you rely very much on situational awareness and an understanding of your battlefield- perhaps moreso than any other class. With that in mind- I imagine it's easy to use a monk in a less than optimal fashion.

I know in particular I've heard of a group with at least three monks at the gaming store near me- two are using some Dragonball supplement that allows them to be Super Saiyan and a Majin Buu while one is a normal Long-Death monk. The Long-Death monk simply because he knows how to be a monk better than them is outperforming them despite their abilities being overpowered as all hell.

CorporateSlave
2018-11-24, 01:09 PM
So to the people who play them: Whats it like playing a monk.
Is there anything you would change? Do you feel good playing it? Are there things you wish you had or things you wish got a little bump?


I've recently tried my first Monk, and went full-blown Kensei; I'm having a blast despite the lack of optimization that Kensei seems to have. Granted, the magic items my DM has given out go a long way towards "improving" the Kensei sub-class in the context of our campaign, but I find the versatility to outweigh the lack of optimization on DPR/etc that earns them so much flak.

I've been able to switch from effective ranged kiting to chasing that last fleeing enemy down, stunning striking to lock down the boss, to using Patient Defense and Agile Parry to tie up anything from a large enemy to a mass of adds while the rest of the party takes care of whatever else. In a pinch I've got the mobility to hustle in a pour a potion down a fallen allies throat from halfway across the battlefield.

Like others have said, it does take some reading of the tactical situation to decide which avenue to pursue on any given turn, but I find the versatility to be quite refreshing after a couple of PC's that while powerfully optimized, really lacked any variety in combat.

Basically if I had to say the one thing that really improves the monk, it would be someone to reliably Haste them...it makes their effective movement so crazy that it makes them even more fun to play!

Citan
2018-11-24, 02:43 PM
I've recently tried my first Monk, and went full-blown Kensei; I'm having a blast despite the lack of optimization that Kensei seems to have. Granted, the magic items my DM has given out go a long way towards "improving" the Kensei sub-class in the context of our campaign, but I find the versatility to outweigh the lack of optimization on DPR/etc that earns them so much flak.

I've been able to switch from effective ranged kiting to chasing that last fleeing enemy down, stunning striking to lock down the boss, to using Patient Defense and Agile Parry to tie up anything from a large enemy to a mass of adds while the rest of the party takes care of whatever else. In a pinch I've got the mobility to hustle in a pour a potion down a fallen allies throat from halfway across the battlefield.

Like others have said, it does take some reading of the tactical situation to decide which avenue to pursue on any given turn, but I find the versatility to be quite refreshing after a couple of PC's that while powerfully optimized, really lacked any variety in combat.

Basically if I had to say the one thing that really improves the monk, it would be someone to reliably Haste them...it makes their effective movement so crazy that it makes them even more fun to play!
1. I really wonder what you mean by that?
Kensei is actually much better than Open Hand for low levels imo. Kensei, contrarily to Open Hands, provides benefits...
- That covers the one blatant weakness Monk shares with Barbarians and to a lesser extent Paladins, which is "I'm a melee guy"! Pick a hand crossbow or a longbow (both later), and contrarily to all other Monks (including Sun Soul which only has 30 feet range, except 4E at high level), you will be as good against flyers/casters/archers as against melee guys.
- That eases much the learning curve by providing a much welcomed, always relevant flat +2 to AC *at absolutely no cost* (there is no reason why you would use versatile weapons two-handed, and per errata you can hold longbow with one hand when not firing arrows).
- That simply improves your Monkiness at a core level, by making weapon attack magical (you won't care what enemy you face), improving damage on one attack (meaning you have less "potential loss" if you use Ki on something else than Flurry of Blows) and then ultimately making you the big boss of all Monks as far as landing Stunning Strike goes (either you maxed DEX, meaning you hit much more often than others, or you maxed WIS, meaning you hit as often as others but your SS sticks more often than others).

2. You can add Warding Bond to that, although it does anti-synergizes with Haste unless the caster is also under its effect (*cough* Divine Soul Sorcerer *cough*) but also simpler Aid or Longstrider, Fly. ;)

KyleG
2018-11-25, 05:33 AM
So the recommendation for monk 1-5 is melee weapon hit and run? or fists? I was thinking billy club type thing but quarterstaff is going to hit better before we run off (3 sectional staff?). As for race...im thinking VHuman with mobility? or maybe martial adept? my other thought is Eldarin Elf with misty step.

SleepIncarnate
2018-11-25, 06:20 AM
Since most monks don't have ranged attacks, the movement, despite being good, can at times be lacking.

This is a stereotype that just needs to die.


This bothers me to no end. Monks are proficient in simple weapons, focus on dexterity, and get extra attack. They're not as dangerous with ranged attacks as they are with melee, but with the defenses you mention (deflect missiles, evasion, patient defense) they're also in a lot less danger.

Even if you don't want to carry a shortbow, every monk should at least have a humble sling.

This.

Now true, none of the simple ranged weapons count as monk weapons for purposes of martial arts damage/bonus action attacks, but that's why there are various monastic order tradition options. Plus the returning ranged attacks at opponents as a reaction.

Add in either a ranged kensei build or sun soul and you could definitely be just as devastating at range as other monks are up close. A kensei can turn a freaking longbow into a monk weapon, granting it the damage progression up to 1D10, as well as all the other martial arts and kensei bonuses. And sun soul is basically a Super Saiyan with beams of light from their hands, which again count for martial arts.

Or instead of taking the attacks, take advantage of the mobility provided by shadows. There are a lot of ways for the monk to counter this issue of "useless at range" that plagues the popular perceptions of them.

But basically, you can pick any kind of martial arts or anime trope and turn it into a monk character. The master archer? The blind swordsman? Both variants on the kensei. Wanna be a Naruto OC? That's shadows. A bender from Avatar? Four elements. Or flavor them off of video games. Ken and Ryu from Street Fighter are both examples of lower level sun soul monks, with their fireballs being the level 3 ability.

I personally love the interest factor in unusual characters. It's become a trope of its own now, but the halfling monk is still great just for the sheer size difference between them and the people they're taking down. Tabaxi make interesting monks, and lend themselves well to shadow monks. But something like a half orc who followed the way of tranquility as a means of combating the orcish rage inside him can be fun and interesting too.

Spore
2018-11-25, 06:23 AM
The class is still underpowered, whether or not the flavor is good @SleepIncarnate

SleepIncarnate
2018-11-25, 06:39 AM
The ability at level 5 to have 4 attacks per round is underpowered? Especially when those attacks end up being 1D10 at higher levels, and with certain subtypes (sun soul, ranged kensei) can even be at range? Even before tacking on other things? Monks may not be the most OP class in the game, but I dunno that I'd go so far as to say underpowered. They're the only class with the option of two attacks per round at level 1 if you're not counting something like the rider on GFB. Even fighters don't pick up action surge until level 2.

I'd say monks are more like paladins, very front loaded, but I'd hardly say underpowered. Just, that sheer level of power they have coming out the door doesn't come every level. They still get some decent stuff, but not sheer amazingness. Probably the last really great ability they get is diamond soul at 14. But most campaigns are wrapping up in the 15-17 range anyways, so that's not as big of an issue.

ImproperJustice
2018-11-25, 06:41 AM
My experience having played with an Open Hand, Shadow, and Kensei has been:

It’s a class built around a lot of “cool”.
Runnig along walls, teleporting behind enemies, and leaping Matrix style across rooftops.

Stunning blows are just heinous and often tacked onto lethal short range barrages.

Bothe the Shadow Monk and the Kensei were lethal ranged combatants. Rarely in harms way and constantly chipping away at the enemy. Eventually they would find a soft spot and unload at short range. Integrating magical weapons into their fighting style seemed to help a lot.

Out of Combat: All were solid scouts. The Shadow was excellent and allowed the whole party to stealth with Pass Without Trace.

Citan
2018-11-25, 06:44 AM
The class is still underpowered, whether or not the flavor is good @SleepIncarnate
Congrats sir, you won the "Most useless and inaccurate post of the day"! :smallbiggrin:
(seriously, putting "Monk" and "underpowered" in the same sentence is hilarious: if a Monk character feels useless in some instance, it's not the class that is underpowered but rather the level of wits player put into that particular fight :smalltongue:)


This is a stereotype that just needs to die.

Now true, none of the simple ranged weapons count as monk weapons for purposes of martial arts damage/bonus action attacks, but that's why there are various monastic order tradition options. Plus the returning ranged attacks at opponents as a reaction.

Yeah. And, in addition to that, on any turn when you don't anticipate to end in close range, you can simply use Javelins or daggers and make thrown weapon attacks. It is not sustainable in consecutive rounds because of the drawing rules (well, you could pick Dual Wielder but that's far too niche for most Monks) but for the occasional need it's far enough.
When you need round after round ranged attack, darts, sling or shortbow depending on context (how far you need to attack, do you expect needing a hand free for Flurry or OA) is quite sufficient.


The ability at level 5 to have 4 attacks per round is underpowered? Especially when those attacks end up being 1D10 at higher levels, and with certain subtypes (sun soul, ranged kensei) can even be at range? Even before tacking on other things? Monks may not be the most OP class in the game, but I dunno that I'd go so far as to say underpowered. They're the only class with the option of two attacks per round at level 1 if you're not counting something like the rider on GFB. Even fighters don't pick up action surge until level 2.

I'd say monks are more like paladins, very front loaded, but I'd hardly say underpowered. Just, that sheer level of power they have coming out the door doesn't come every level. They still get some decent stuff, but not sheer amazingness. Probably the last really great ability they get is diamond soul at 14. But most campaigns are wrapping up in the 15-17 range anyways, so that's not as big of an issue.
1. As much as I appreciate Monk otherwise, that bit is wrong. Any character can go dual-wield to get two attacks at level 1, one with action, one with bonus action, like Monk.
The difference is that it's built-in in Monk whereas others have to cope with equipment restriction (light weapons) and a Fighting Style to reach the same efficiency. :)

2. I strongly disagree with that: Empty Body is the best feature any martial can hope for: non-concentration, 1mn, invisibility (meaning advantage against most creatures, disadvantage for them attacking you, flat-out immunity against a majority of spells unless caster has buffed himself with See Invisibility or True Seeing) and resistance to all damage (in the occurence that, in spite of proficiency + reroll, you did suffer damage).
It makes any Monk as resilient as Bear Barbarian and Paladin combined. Open Hand especially,
thanks to the lvl 6 "action to regain ~54 HP", can survive unanticipated bad round.
It also makes any Monk extremely sneaky. Shadow especially, combining that with Pass Without Trace, can simply best even Rogues in situations where being both unseen and unheard is a flat requirement.
Open Hand especially, combining that with one of the top 5 features of the whole game (Quivering Palm), can make a better assassin than even Assassins themselves.
With a Ring of Spell Storing, you can be the holder of a powerful party/ally buff or enemy debuff (*cough* Haste / Circle of Power / Polymorph / Fly / Elemental Weapon *cough*) with the overall best chance of maintaining concentration for the aforementioned reasons.

sophontteks
2018-11-25, 09:50 AM
The class is still underpowered, whether or not the flavor is good @SleepIncarnate
It's not under-powered. You just haven't learned how to play it.

If you play a monk like a fighter it will be under-powered. If you actually use the monks mobility to flank and CC key targets with stunning fist, the monk will take the spotlight. You're bashing a class that can kite any melee enemy, has resistance to everything ranged, and has the best single-target CC in the game. That's not flavor. That's missed opportunity.

Sahe
2018-11-25, 10:14 AM
But basically, you can pick any kind of martial arts or anime trope and turn it into a monk character. The master archer? The blind swordsman? Both variants on the kensei. Wanna be a Naruto OC? That's shadows. A bender from Avatar? Four elements. Or flavor them off of video games. Ken and Ryu from Street Fighter are both examples of lower level sun soul monks, with their fireballs being the level 3 ability.


Strongly disagree on that point. 4 Elements as it is in the PHB is utter garbage at emulating a bender. A thematic Monk/Sorcerer or even pure Sorcerer with thematically picked spells is MUCH better at emulating that. 4 Elements monk just get's too few abilities that cost too much and essentially lose all flavor of their subclass once they run out of Ki...which happens fast. Another problem is that they never got their options expanded and they only get a very limited number of unique tricks to them.

SleepIncarnate
2018-11-25, 10:17 AM
2. I strongly disagree with that: Empty Body is the best feature any martial can hope for: non-concentration, 1mn, invisibility (meaning advantage against most creatures, disadvantage for them attacking you, flat-out immunity against a majority of spells unless caster has buffed himself with See Invisibility or True Seeing) and resistance to all damage (in the occurence that, in spite of proficiency + reroll, you did suffer damage).
It makes any Monk as resilient as Bear Barbarian and Paladin combined. Open Hand especially,
thanks to the lvl 6 "action to regain ~54 HP", can survive unanticipated bad round.
It also makes any Monk extremely sneaky. Shadow especially, combining that with Pass Without Trace, can simply best even Rogues in situations where being both unseen and unheard is a flat requirement.
Open Hand especially, combining that with one of the top 5 features of the whole game (Quivering Palm), can make a better assassin than even Assassins themselves.
With a Ring of Spell Storing, you can be the holder of a powerful party/ally buff or enemy debuff (*cough* Haste / Circle of Power / Polymorph / Fly / Elemental Weapon *cough*) with the overall best chance of maintaining concentration for the aforementioned reasons.

Oh, I don't disagree. Maybe I should be more specific that the last great thing for most every campaign is diamond soul. I can think of great uses for empty body as well, but those are going to depend on the campaign, the rest of the party, etc. and may not work as well in a heavy combat setting. Sure, when combined with specifically the empty hand heal, it's nice. The other idea of going invis while holding onto concentration for a buff for the party is nice, but by level 18, there's enough things with blindsense, see invis, or whatever else that you're basically either having to take yourself out of the fight completely to guarantee concentration. Especially if your DM rules that it works per invisibility and not greater invisibility, and thus breaks as soon as you attack. Outside of combat, still great, but in combat, not so much.

EDIT BELOW:


Strongly disagree on that point. 4 Elements as it is in the PHB is utter garbage at emulating a bender. A thematic Monk/Sorcerer or even pure Sorcerer with thematically picked spells is MUCH better at emulating that. 4 Elements monk just get's too few abilities that cost too much and essentially lose all flavor of their subclass once they run out of Ki...which happens fast. Another problem is that they never got their options expanded and they only get a very limited number of unique tricks to them.

I don't disagree that Four Elements is the weakest of the monastic ways, and flavor for it can be done better with other classes. But this is a thread specifically about monks, and it honestly the closest thing I could think of as an example for what a 4E monk would be in anime/movie tropes. Other ways do what 4E wants to do but better, such as sun soul with the fire magic (though most of it is actually light/radiant except the 6th level burning hands, but even that's better with being scaleable). I was trying to fit all of them in there though, to give OP some feel for options. But yes, 4E sucks.

sambojin
2018-11-25, 10:39 AM
How do monks feel? Kinda like warlockish/BMish-moon-druids, but better at combat while being worse at casting.

Druids can do WAY more shenanigans, magical or otherwise, but are so very limited in what they'll do the next turn after that and in the attacks/actions/thingies they'll bring/do each turn after deciding on how they'll approach that particular encounter. It's kind of like a rogue -> fighter comparison melee/encounter wise. Neither's horrible, but fighter is better in general in fights and doesn't really commit to a playstyle to be effective in them, while a rogue can approach a huge variety of situations in many different ways, but it tends to just get down to sneak-attack rolls in combat (ala, using whatever your current druid wildshape shtick is compared to a monk's "fighter" tactical flexibility with "warlock EB/flurry" backup).

Citan
2018-11-25, 03:10 PM
I don't disagree that Four Elements is the weakest of the monastic ways

But yes, 4E sucks.
Well, 4e is actually the strongest in several areas, you know, from level 6-7 to level 17 (Quivering Palm simply cannot be beaten). And from levels 3 to 6, it can be better than Open Hand in melee situations, and more or less equal to Sun Soul: Fangs of the Fire Snake give you very simply magical damage at 15 feet reach for just one ki. Meaning you can make melee attacks without getting into OA. And it can stack with Stunning Strike as a consequence.

People just cringe, for good or not, about the heavier ki cost it usually gets compared to other. Which can be true often, or not, depending on each one's wits, party tactics and opponents (for example: theorycrafting, Shadow has the best cost to benefit ratio since casting spells that last a minute each for 2 ki: but in fight, you cannot always avoid a concentration-breaking hit especially since you probably don't have great Constitution, nor proficiency).

But he is the only one that can really do different things in different ways, and although its abilities are few, they are all good. More importantly, it's the only one that allows you, should you wish so, to get really different playstyles from one level to another.

And when you get Fly and Fireball, you are simply the best Monk as far as versatility goes. :) In fact, just Fly makes you simply the best at being a Monk since flying speed can be stacked upon your Monk bonus speed.

So, no 4E doesn't suck. It's just different to play at low levels and tend to use abilities more in a nova fashion rather than through small low-cost uses regularly.

Sahe
2018-11-25, 04:28 PM
Well, 4e is actually the strongest in several areas, you know, from level 6-7 to level 17 (Quivering Palm simply cannot be beaten). And from levels 3 to 6, it can be better than Open Hand in melee situations, and more or less equal to Sun Soul: Fangs of the Fire Snake give you very simply magical damage at 15 feet reach for just one ki. Meaning you can make melee attacks without getting into OA. And it can stack with Stunning Strike as a consequence.

People just cringe, for good or not, about the heavier ki cost it usually gets compared to other. Which can be true often, or not, depending on each one's wits, party tactics and opponents (for example: theorycrafting, Shadow has the best cost to benefit ratio since casting spells that last a minute each for 2 ki: but in fight, you cannot always avoid a concentration-breaking hit especially since you probably don't have great Constitution, nor proficiency).

But he is the only one that can really do different things in different ways, and although its abilities are few, they are all good. More importantly, it's the only one that allows you, should you wish so, to get really different playstyles from one level to another.

And when you get Fly and Fireball, you are simply the best Monk as far as versatility goes. :) In fact, just Fly makes you simply the best at being a Monk since flying speed can be stacked upon your Monk bonus speed.

So, no 4E doesn't suck. It's just different to play at low levels and tend to use abilities more in a nova fashion rather than through small low-cost uses regularly.

Honestly, I don't care so much about effectiveness as I care about the fantasy it's trying to sell and to me, that fantasy and the inspiration (and probably everyone else who see's this subclass) is pretty much Avatar. And that's where this subclass fails because you know so few techniques and can only use them so few times. A 4 Elements monk doesn't feel like a master of the Elements or even one Element. Like you can't even properly built around a single element. 4 Elements Monk for me doesn't suck because it's weaker than the other Monk subclasses, but because it fails to deliver what I want from it. To be fair, the Monk is probably a poor Chassis for a Avatar style bender of the Elements but that's an entirely different issue.

sambojin
2018-11-25, 10:41 PM
4e monk does feel way too limited compared to just grabbing a level or three of sorceror or wizard or druid or cleric or warlock or bard or pally or fighter or nearly anything with actual magic/combat.

I'd give them a "you get an extra Elemental Discipline at lvl3, because you don't have enough of them". And a "you get 1/2 your Monk level worth of Ki points *per day* to utilize in non-4e Ki uses, as a side Ki-pool, so you can still do normal Monk stuff. Ask your DM whether you round up or down". BTW, it was a typo, so "every Elemental Discipline is actually *4 levels lower on its prerequisites down to a minimum of lvl1*. You can also *expend one extra ki point on them than listed on the 4e-Monk-level/maximum-Ki-use chart*. Sorry, our editor forgot how to math and how d&d works", because we don't care if you don't have enough Ki for them yet or to use them regularly, but you can just do you and choose what you want at an appropriate level for those kinds of abilities.

That'd sort the problem. Would it make them powerful? Kinda. But honestly not that good anyway. Good, but not OP. Not compared to other classes and builds available, MC or not. You'd be like a punchy martial warlock, with a few spells, but nothing powerful, and a bit of extra movement. Might even take the prerequisites down by 5lvls, but that may be too much. Don't want any complaints about power creep destroying an overly resource intensive class. Considering what a level or two of warlock or moon druid can give you :)

I wonder if we'll get a Planeshift: Gamemaster's Guide to Kamigawa that remedies fighters (not really needed) and rangers and monks as well? Because, WotC and MtG...... The Book of Weeaboo Fighting Techniques Restored: Return to OP Ninja-land! Feats and disciplines and totally not admitting ranger needs an official cool-factor add-on. With more 4e than you can shake your avatar at!

Malifice
2018-11-25, 11:14 PM
Have had great success with a Monk 5 (Kensai) BM Fighter 3.

A stunned critter automatically fails Str and Dex saves. It's rather funny to stun a monster then spam a tripping or disarming strike it cant save against.

Am also considering 3 levels of Hunter ranger for 3 x Hunters Marks per day plus the extra damage from Colossus slayer.

That said, DPR is more than adequate at present so sticking with Monk for all the other goodies it offers plus the extra Ki points is probably how I'll roll from here on in.

Spriteless
2018-11-25, 11:28 PM
In the 5e game I run, the monk stunlocks the scariest guy and the rogue gets easy sneak attack an the stunned guy. The party either follows suit, or take out a few mooks per round. Depends on who would make the better prisoners. But they are free to decide that, instead of playing by the rules Scary Guy counts on.

So, helpful in a fight, everyone gives props to him, but not as likely to get the finishing blow.

This monk seems to lack the peaks and valleys both that the other characters have. The immunuties to random statuses add to the lack of valleys. I guess if I ever stole the party's equipment, then the monk would shine.

If the party has another short rest class, like a warlock, you will be full up enough on ki points to use the abilities most of the time, but... it isn't nova, but lock down. Do you like blue/white decks? I feel like I'm playing against one.

He can run up walls. Do you know how many modules are weird when the party comes in through the roof? It's only the ones with buildings ah ha ha.

sophontteks
2018-11-25, 11:36 PM
He can run up walls. Do you know how many modules are weird when the party comes in through the roof? It's only the ones with buildings ah ha ha.
Walls are pretty common in dungeons. And the name of the game is Dungeons and Dragons afterall.:smallbiggrin:

sambojin
2018-11-25, 11:38 PM
@Malifice
Yeah. Monks are nice MC'd, even if you do go MAD, strange armour'ish and kinda weird on how you do stuff. Short rest resources are always fantastic, and having BM dice, AS, Ki, etc, would work great.

Single classed, they're kinda average. Not weak by any means, but neither is any other class in 5th edition if played to its strengths.

4eMonk is an outlier due to how resource intensive they are single classed with very limited "not-always-on bonuses", while still having somewhat limited abilities later on, yet it's not entirely terrible. It's just not OMFG, I just hit lvl3/5/6/7/9/10/11 levels of good that so many other classes get.

As a dip, they're semi-pointless. As a single-class, they're just pretty weak feeling compared to other Monk subclasses or almost anything with "abilities/bonuses". There's few other classes that you couldn't flavour or play towards doing 4e stuff that wouldn't feel more fluid and flexible than them, even knowing all the basic Monk abilities coming through later on (lvl10-15, and with a big pool of short rest resources).

sambojin
2018-11-25, 11:43 PM
He can run up walls. Do you know how many modules are weird when the party comes in through the roof? It's only the ones with buildings ah ha ha.


Walls are pretty common in dungeons. And the name of the game is Dungeons and Dragons afterall.

Two levels of Moon Druid. Well, any Druid. Giant Wolf Spider for normal(1/4CR, medium) or Giant Spider for moon (1CR, large, easily rideable). Welcome to unlimited wall walk/spider climb. Better than any monk.

sophontteks
2018-11-25, 11:52 PM
Two levels of Moon Druid. Well, any Druid. Giant Wolf Spider for normal(1/4CR, medium) or Giant Spider for moon (1CR, large, easily rideable). Welcome to unlimited wall walk.
That's limited to 2 per short rest actually. :smallamused:

The monk is retaining his form, using no abilities nor bonus actions, and its all a part of his already high movement. Welcome to actual unlimited wallwalk.

Call me when a druid can run across a wall and deliver 3 saves vs. stun in one round.

sambojin
2018-11-26, 12:09 AM
Hours of wall walking, free HP, blindsight, a ranged restrain (OK, crappy DC, but action-use removal). At level 2.

Also, it's not the only thing they do by level 9. By then they have uber summons, talking intelligent pets, massive f'you tank heals, big DoT spells, on-call flying (that can carry others), perhaps some stupid wildshape forms/always accessible abilities/wazoo spell-lists, and kinda free rein on how to approach matters.

It's not Druid > Monk, they're very different. But saying "I can wall-walk for cheap and punch people into comprising positions by lvl9" is not a good yardstick of what other classes can do at a similar level. Druids aren't actually that powerful either.

Malifice
2018-11-26, 12:10 AM
@Malifice
Yeah. Monks are nice MC'd, even if you do go MAD, strange armour'ish and kinda weird on how you do stuff. Short rest resources are always fantastic, and having BM dice, AS, Ki, etc, would work great.

It's not MAD though. Fighter and Ranger qualify with Dex (and Wisdom for the Ranger which you'll also have) and provide a nice DPR boost to the Monk. Rogue is also a nice way to go for extra skills plus expertise, also a nice boost to DPR with sneak attack, and the always handy Cunning action.


Single classed, they're kinda average. Not weak by any means, but neither is any other class in 5th edition if played to its strengths.

No, they're really not. Stunning fist is arguably among the best class features in the entire game (up there with +Cha to saves, proficiency in all Saves, or high level spell casting).

In a core game (one averaging a median of around 6 or so medium-hard encounters per long rest, with 2-3 short rests in that time) Monks are fantastic. Stunning fist lets them lock down priority targets (spell casters, heavy hitters, leader types) for a round or two - which is all the time you need to completely turn an encounter.

I've seen DMs really struggle in parties with Monks, and many a DM bemoan Stunning fist as being broken after seeing a Monk shut down a BBEG or scary CR'd monster by simply walking over and spamming Stunning fist on it, totally ending the encounter before it begins and stun-locking a priority target for a few rounds.

Monks are the most mobile class in the game, come with the best defences of any class in the game (eventually having all Save proficiencies, plus the ability to spend a Ki point to re-roll a save, plus a host of immunities) have good DPR (not Paladin or GWM Barbarian good, but more than adequate for frontline duties) and have the best at will Save or Suck class feature in the game (stunning fist) that is great for burning Legendary resistances (no monster wants to be stunned for a whole round) and for locking down priority targets.

They also come with great utility out of the box.

Monks can boost DPR simply by picking up a magic weapon just like any other class (a flame tounge short sword is a nice way to go as are any of the big ticket magical short swords, as is a Staff of striking), Bracers of defence work wonders on them also.

The only bad thing about Monks is once you figure out how to best use one as a player (identify target and isolate the nastiest creature in the encounter with Stunning fist, stun lock it and then wait for the Cavalry to arrive, and dont waste your time dealing with mooks) they become kind of repetitive:

DM: 'You see a figure in robes holding a staff starting to cast a spell, while 6 henchmen in chain shirts draw weapons and advance. Near the robed figure is a big burly human holding a greataxe shouting commands and twirling his axe with obvious skill. Roll initiative'
Monk player: [Already thinking about how to close with the spellcaster and past the mooks, before the spell goes off on the casters turn, while also checking to see how many Ki points he has left this SR].

Once you get the hang of them, they're effective if a little boring.

Id really like more options with Ki (martial arts style feats perhaps that add extra Ki point options?].

sambojin
2018-11-26, 12:27 AM
Nope. Certainly not the most mobile. There's shadowy teleporters, casters that get non-AoO movement on cast, moon druids (flyby and grapples/dive attacks from a Quetzlcoatlus is *mobile*, even with crappy to-hit), rogues, etc, are all right up there.

Not saying Monks aren't good at some stuff. Certainly not. They're fantastic at some stuff, and pretty good at some other stuff. But saying "yep, they're the best at that kinda stuff" is a bit silly on all the class archetypes, races, backgrounds and even single classed builds that are about now.

They're "good in the can, because they do what it says on the can (except 4e)". But they're not the "best", because other stuff does that and more. This doesn't make them bad. They're very good at what they do, and as mentioned, it requires a strategic mindset and a thorough knowledge of the situation and your skills and how to best utilize them for you to truly show a Monk's abilities off. And then they're astounding.

But they're not a brain-dead "don't worry, we're good. I'm good. Are you good? If not, I'll just do something else off my massive list of options in any particular situation."

Focused? Yes. And good at that.

Good good? Nup.

Stunned? Blinded? Restrained? Incapacitated? Paralyzed?

Yep, they CAN spam stun. And stunned *is* good. But it's not exactly the only available condition to put onto 1-2 big creatures that can make the rest of the party pop mega-damage over the next turn or so. And other classes can get some of these in spades as well, at a pretty comparable DC, multi-tap them onto the enemy in some cases, or AoE them onto many of them. So, it's *good*. But, ummm, yeah. Well done. Monks do *a thing*. Lol.

NecessaryWeevil
2018-11-26, 12:53 AM
Typically, their ascetic training conditions them to feel very little.

Astofel
2018-11-26, 12:55 AM
The monk will most likely have the highest move speed in the party after 2nd level, and they can burn ki to dash or disengage whenever they like. And if you're in a game that makes good use of short rests running out of ki isn't a worry; my 5th level monk was using ki almost every round and I never felt like I was in danger of running out. If you pick up the mobile feat you don't even need to burn ki, and can instead potentially attack 4 different creatures in one turn, provoking no AoOs, and even stun them if you have the ki to burn. Even druids are limited in mobility by the form they're in, and often the most mobile form isn't the best one in a fight. After all, if you're currently a giant spider, then you're not a cave bear. To me monks definitely have the highest potential mobility, more or less at-will.

Sure, rogues might have the bonus action Disengage at-will, but they can't kite like the monk can. Most monsters can just use their move to catch up to the rogue, or they can Dash if the rogue Dashes, and now the rogue is left in the same position they were in before. Monks can outpace most monsters, and still get their attacks off while kiting. Other classes can teleport, but a well-placed shadow monk can do it basically at-will, and gain advantage from it to boot. About the only thing the monk can't do is fly, but very few classes get at-will flight anyway.

I think it's also worth pointing out that being a 10th level monk is one of the few ways to get an always-on flat-out immunity to a damage type. The only other way iirc is to be a 17th level forge cleric. I don't know how common poison damage is in the MM, but I bet it feels pretty great to stand in the breath weapon of a green dragon and laugh.

LudicSavant
2018-11-26, 01:18 AM
Nope. Certainly not the most mobile.

Sambojin appears to have the right of it. Monks are quicker than your local Champion Fighter (outfitted in his Snail Mail), but they're not the fastest around.

Consider Vengeance Paladins sharing haste with their Pegasus mounts (for 540 foot move speed)... and then being able to teleport and take their steed with them. Or Kitelocks. Or Bards!

A monk has, what, a 30 foot move speed bonus by level 18? And the ability to burn their bonus action (and with it a good chunk of their offensive contribution for the round) and a ki point to dash or disengage? And some modest vertical options.

A Monk's walking isn't even supernaturally fast. As in, a level 20 Monk would lose in the Olympics. 60 feet / 6 seconds is only what, about 6.8 miles an hour? Dashing is only around 13.6? I guess you could Dash with your Action and with your bonus action / ki to get 180 feet in a round, but that's only about 20.5 mph!

Usain Bolt runs at 28 miles per hour. Why can't I run at least as fast as an Olympic athlete while burning ki and all of my actions on movement at tier 4?

djreynolds
2018-11-26, 01:21 AM
It's great class, and the archetype are really just icing.

At 4th level, grab the mobile feat, allowing you to use your huge movement to strike and move. This will help save KI normally used for patient defense and step of the wind, for FOB and stunning fist.

You will begin with a 16 dexterity, so with a short bow you have ranged attack, and if you a wood elf, grab a longbow. +3 to hit, only a ranger or fighter with archery style has better.

All the archetype are good, even 4 elements is better than you might think as it give you something to do you normally couldn't.

The basic monk is great, grab the mobile feat early. Being able to extricate yourself out of melee without getting hit, is as good as a high AC.

Grab a spear or staff, short sword, and a bow. That's your ranged attack, a bow. 80ft for a short bow, 150ft for a longbow

As for multiclassing, cleric is sweet even just for the bless spell. Fighter for archery style, or 2 of ranger for hunters mark and archery style

For ASI, mobile, 2 to max dexterity.
After 12th level this leaves 2 more feats, could go wisdom twice for AC and DC, or lucky or sharpshooter or tough.

Mobile feat right away at 4th, or 1st if vhuman, and get a bow.

sambojin
2018-11-26, 01:22 AM
Or punch someone right beside me, with nearly knockout blows, more than once every 1.5sec? Too slow, I reckon. Totally unrealistic.... Lol

sambojin
2018-11-26, 01:29 AM
I think it's also worth pointing out that being a 10th level monk is one of the few ways to get an always-on flat-out immunity to a damage type. The only other way iirc is to be a 17th level forge cleric. I don't know how common poison damage is in the MM, but I bet it feels pretty great to stand in the breath weapon of a green dragon and laugh.

Land Druid, lvl10, always.

Moon Druid, lvl10, Elemental Wildshape (all are immune to poison), uses both WS charges, but 5hr runtime. Yes, even moons like short rests.

Spore Druid. Maybe? Lvl14. Really weirdly worded. It reads like its poisoned condition immunity, not actually poison immune.

Just saying..... That's why I compared the two classes. One's a BM fighter/warlock that's REALLY good at some stuff (Monk), the other's kind of like a versatile rogue with other stuff that's not "always on" but with a grab-bag of tricks for any situation (Druid, does combat simply, but is kinda versatile in lots of other stuff).

SleepIncarnate
2018-11-26, 02:32 AM
As a dip, they're semi-pointless.

Druids would like to have a word with you... Especially the moon druids wanting to add their wisdom modifiers to the AC of their shifted forms. And caster druids wanting that as well for even their caster forms.

LudicSavant
2018-11-26, 02:37 AM
I think it's also worth pointing out that being a 10th level monk is one of the few ways to get an always-on flat-out immunity to a damage type. The only other way iirc is to be a 17th level forge cleric.

Other ways off the top of my head just for poison immunity:
- You can make the whole party immune for 24 hours at level 11 (1 level after the Monk gets immunity) by using Heroes' Feast, which every Cleric and Druid knows.
- Yuan-Ti Purebloods (PC race in Volo's Guide to Monsters) get poison immunity.
- Level 10 Land Druids get poison immunity.
- Shapechangers (like Moon Druids) can get immunity (it's the most common damage immunity in the Monster Manual).
- The Periapt of Proof Against Poison gives immunity to whoever wears it, and doesn't even take attunement.

Some more obscure stuff too:
- Planar Vitality (optional rule in the DMG) grants it when you're on Arcadia.
- The Boon of Perfect Health will give you immunity.
- Eye and Hand of Vecna grants Poison Immunity.
- Axe of the Dwarvish Lords grants Poison Immunity.

Also worth noting is that resistance to Poison is generally easier to get than other resistances. For example Dwarves, Stout Halflings, and Green Dragonborn have Resistance to poison. And a 2nd level spell (Protection from Poison) gives you resistance to poison for an hour without Concentration. And Infernal Constitution (the tiefling half-feat) gives you resistance to Poison damage.

Malifice
2018-11-26, 02:46 AM
Nope. Certainly not the most mobile. There's shadowy teleporters,

You mean Shadow monks?


casters that get non-AoO movement on cast,

A few times per day that chews into other resources used for offence.


moon druids (flyby and grapples/dive attacks from a Quetzlcoatlus is *mobile*, even with crappy to-hit),


Robbing Peter to pay Paul.


rogues

Monks are better.

LudicSavant
2018-11-26, 03:44 AM
To be a candidate for "most mobile," you'd have to answer questions like:

> Can you move over 500 feet a round while keeping your actions free?
> Can you teleport? How far? In how many situations? Can you do it when it's not your turn?
> How much can you move when it's not your turn?
> How good are you at flying? Can you do things like go 600 feet a round for 8 hours?
> How convenient is it for you to walk through solid walls, floors, or ceilings? Can you attack people effectively from the other side?
> Can you engage an enemy this turn from literally at least one mile away?
> How's your planar travel looking? Can you chase someone who's swapping between layers of existence?
> How much offense do you have to sacrifice for your mobility?

...For starters.

Here's Monk mobility in a nutshell: They get a 30 foot bonus to their movement speed by level 18, and they get the ability to dash as a bonus action that eats ki... and that bonus action basically means cutting their attacks in half so it's not like they're just tossing it out there like Rogues are (most of their damage is rolled up in their Sneak Attack and off-turn Sneak Attack). And they can run on vertical surfaces (which basically is about as good as having one of the benefits from the Athlete feat). That's pretty much it if you're an Open Hand Monk. Shadow Monks at least get a decent bonus action teleport.

It's better mobility than some other martials get, but the best? It takes a bit more than that for such a title.

Astofel
2018-11-26, 05:08 AM
Other ways off the top of my head just for poison immunity:
- You can make the whole party immune for 24 hours at level 11 (1 level after the Monk gets immunity) by using Heroes' Feast, which every Cleric and Druid knows.
- Yuan-Ti Purebloods (PC race in Volo's Guide to Monsters) get poison immunity.
- Level 10 Land Druids get poison immunity.
- Shapechangers (like Moon Druids) can get immunity (it's the most common damage immunity in the Monster Manual).
- The Periapt of Proof Against Poison gives immunity to whoever wears it, and doesn't even take attunement.

Some more obscure stuff too:
- Planar Vitality (optional rule in the DMG) grants it when you're on Arcadia.
- The Boon of Perfect Health will give you immunity.
- Eye and Hand of Vecna grants Poison Immunity.
- Axe of the Dwarvish Lords grants Poison Immunity.


Note that I specified always-on. That rules out Heroes' Feast (sure it lasts 24 hours, but it's not cheap), Shapechangers, and Planar Vitality. Everything else but the land druid requires a generous DM. Most DMs I know agree that the yuan-ti is too strong a race and wouldn't allow it, and the others are all not guaranteed to happen in any campaign, particularly the artifacts.


To be a candidate for "most mobile," you'd have to answer questions like:

> Can you move over 500 feet a round while keeping your actions free?
> Can you teleport? How far? In how many situations? Can you do it when it's not your turn?
> How much can you move when it's not your turn?
> How good are you at flying? Can you do things like go 600 feet a round for 8 hours?
> How convenient is it for you to walk through solid walls, floors, or ceilings? Can you attack people effectively from the other side?
> Can you engage an enemy this turn from literally at least one mile away?
> How's your planar travel looking? Can you chase someone who's swapping between layers of existence?
> How much offense do you have to sacrifice for your mobility?


Is this even 5e we're talking about? Most of this list seems like a stretch for anyone to accomplish. I'll go over all of these, but I'm AFB so feel free to school me if I'm wrong.
> As far as I know the only way to do this is the Wind Walk spell, which restricts what you can do with your actions. Dimension Door as well, but that uses your action.
> Several classes can teleport, but iirc most of them need spell slots to do so. The shadow monk can go 60 feet as a bonus action, the only requirement being they're in at least dim light, which shouldn't be hard to find, especially in a dungeon environment. You know, the thing the game was named for. Off-turn teleporting is something I've only seen the mystic do, outside of readying an action.
> I don't recall any classes that can do this reliably, but I'm probably missing something
> I think the only way to fly that fast for that long is Wind Walk
> Passwall or Stone Shape sounds like the best way to do this, and if one party member can do that everyone benefits. if only one party member can go through walls then it's less useful because they get stuck by themselves with an enemy and no way to quickly get help from their buddies.
> I don't actually know how many feet are in a mile, but Eldritch Sniper sorlocks are best at this. I don't see it as useful unless the whole party can also do it though.
> I believe high-level monks can go to either the astral or ethereal plane. Otherwise, if one party member can Plane Shift, they all can.
>Monks sacrifice none of their offense for mobility.

Personally I don't know why 500-600 feet seems to be your benchmark move speed. If one party member races off to go fight something that far away, it's going to take a while for their buddies to catch up and lend them a hand. When I call monks mobile I'm talking about how efficiently they can move their mini about on a battle map a DM might plausibly use. Every monk is easily capable of getting behind enemy lines to target squishies in the back and getting out again, or Disengaging from their current opponent to go feed a potion to a downed party member or distract the guy they were fighting, and for the barest of resources too. All I really need for mobility is the ability to avoid AoOs reliably and a decent move speed, 100 feet/round is the highest you're ever likely to need imo, and it's trivial to build a monk capable of that.

Malifice
2018-11-26, 08:10 AM
Note that I specified always-on. That rules out Heroes' Feast (sure it lasts 24 hours, but it's not cheap), Shapechangers, and Planar Vitality. Everything else but the land druid requires a generous DM. Most DMs I know agree that the yuan-ti is too strong a race and wouldn't allow it, and the others are all not guaranteed to happen in any campaign, particularly the artifacts.



Is this even 5e we're talking about? Most of this list seems like a stretch for anyone to accomplish. I'll go over all of these, but I'm AFB so feel free to school me if I'm wrong.
> As far as I know the only way to do this is the Wind Walk spell, which restricts what you can do with your actions. Dimension Door as well, but that uses your action.
> Several classes can teleport, but iirc most of them need spell slots to do so. The shadow monk can go 60 feet as a bonus action, the only requirement being they're in at least dim light, which shouldn't be hard to find, especially in a dungeon environment. You know, the thing the game was named for. Off-turn teleporting is something I've only seen the mystic do, outside of readying an action.
> I don't recall any classes that can do this reliably, but I'm probably missing something
> I think the only way to fly that fast for that long is Wind Walk
> Passwall or Stone Shape sounds like the best way to do this, and if one party member can do that everyone benefits. if only one party member can go through walls then it's less useful because they get stuck by themselves with an enemy and no way to quickly get help from their buddies.
> I don't actually know how many feet are in a mile, but Eldritch Sniper sorlocks are best at this. I don't see it as useful unless the whole party can also do it though.
> I believe high-level monks can go to either the astral or ethereal plane. Otherwise, if one party member can Plane Shift, they all can.
>Monks sacrifice none of their offense for mobility.

Personally I don't know why 500-600 feet seems to be your benchmark move speed. If one party member races off to go fight something that far away, it's going to take a while for their buddies to catch up and lend them a hand. When I call monks mobile I'm talking about how efficiently they can move their mini about on a battle map a DM might plausibly use. Every monk is easily capable of getting behind enemy lines to target squishies in the back and getting out again, or Disengaging from their current opponent to go feed a potion to a downed party member or distract the guy they were fighting, and for the barest of resources too. All I really need for mobility is the ability to avoid AoOs reliably and a decent move speed, 100 feet/round is the highest you're ever likely to need imo, and it's trivial to build a monk capable of that.

Yeah; spells.

Wizards are the most mobile because Teleport.

Although of course Monks can do that too.

Dark Schneider
2018-11-26, 09:37 AM
The monk is more funny than looking for the ultimate character. Gets many interesting abilities, the Open Hand must be really funny but for its lvl 17 ability, something like a Fist of the North Star, the other are useful but not so great to use. In the case of Shadow Monk AKA Ninja, is the reverse, all its abilites but the lvl 17 are really funny to use.
The Four Elements monk is very underrated, and seems very interesting to use the ki for casting spells, with a great option I think much missed, you upgrade a spell 1 level per 1 ki point you add, so you can cast Hold Person using 3 ki points, but using only 4 ki points you could cast it as 3rd lvl to hold 2 tagets, and 5 ki points to hold 3 targets...
So you not only get more options to use the ki, but also your existing ones get much more power.

Misterwhisper
2018-11-26, 12:22 PM
My issues with monks is mainly thus:

1. They have very little variance.
Every monk that is built to be any good at being a monk will start with stats that are almost exactly the same. They will all fight with either a monk weapon or unarmed, but none of their weapons qualify for the broken combat feats except quarterstaff but they already get a bonus attack so really they all just attack the same. Their AC's, to hit, gear, damage, and tactics will almost be identical. Only real variety is from a Kensei, and well they kind of suck, but could be very easily fixed with one small change, let them take unarmed strike as a kensei weapon.

They can not use shields or armor, not like others who might not be proficient, they simply can't use them so there will be almost no variance in defensive gear.
subsequently due to not being able to use them, their ceiling of AC is the lowest in the game. Even a wizard with one level of fighter/paladin/certain clerics can walk around with medium armor and a shield for a base AC of 19 with no magic items, a monk would have to max one stat and have another at 18 to get that, and can never have more than a 20 without the use of magic gear or spells.
With magic gear anyone with proficiency can pick up some magic half-plate and magic shield and walk around with an ac of 23 - 26, a monk will never have more than a 23, and that is if they multi class and have bracers of armor.

2. There are no magical fist weapons.
I understand that the people at WOTC do not really care about martial gear or equipment it is just an afterthought but the idea that there are no magical gloves/gauntlets/whatever is just bad design and insulting. Essentially, don't bother playing a monk in a by the book game or AL, when everyone else has magical +1 or 2 gear, you will get nothing. If you do get a magical monk weapon, it only covers half your attacks in a round. You still have to attack unarmed with your bonus actions.

3. They need so many stats that you will not really get to customize much with feats unless you are a variant human. Even then, there are not that many great monk feats.

DaveOfTheDead
2018-11-26, 12:42 PM
I play a halfling sun soul monk (subclass is from Xanathar's Guide) and I love it. I just reached level 9 where I can run along walls or liquid surfaces. My AC is 19 with a 20 dex and 18 wis. Monks get front loaded pretty good, so once they reach like level 6 it seems like it slows down a bit, but they still get something every level.

As a DM, I don't usually get to play a long term character, so having this monk is so much fun.

LudicSavant
2018-11-26, 01:39 PM
Note that I specified always-on. That rules out Heroes' Feast (sure it lasts 24 hours, but it's not cheap), Shapechangers, and Planar Vitality. Everything else but the land druid requires a generous DM. Most DMs I know agree that the yuan-ti is too strong a race and wouldn't allow it, and the others are all not guaranteed to happen in any campaign, particularly the artifacts. Your statement was that, if you recalled correctly, there were only two ways to get always-on elemental immunities in the game: 17th level Forge Cleric and 10th level Monk. I was merely informing you that this recollection is not correct.

Incidentally, you were also including immunities to things other than poison on that list, which would make a full correction list longer (for example, Storm Sorcerers get immunity to Thunder damage and Lightning damage permanently).


Is this even 5e we're talking about? Yes, obviously?


I'll go over all of these, but I'm AFB so feel free to school me if I'm wrong. Okay! :smallsmile:


>Monks sacrifice none of their offense for mobility. If losing half your attacks (by foregoing Flurry of Blows for Step of the Wind or the like) doesn't count as losing some of your offense, I'm not sure what does.


> I don't actually know how many feet are in a mile, but Eldritch Sniper sorlocks are best at this. I don't see it as useful unless the whole party can also do it though.
There are 5280 feet in a mile. An example of a way to attack at this range would be Meteor Swarm (it has a range of 1 mile by default).


Off-turn teleporting is something I've only seen the mystic do, outside of readying an action. Examples of ways to do it: Fey Warlock (one of the good choices for high mobility kitelocks). Wizard with Contingency.

Incidentally there's also a fair bit of off-turn movement that isn't teleporting, too, such as a Vengeance Paladin (can move half their speed as part of a reaction).


> Passwall or Stone Shape sounds like the best way to do this, and if one party member can do that everyone benefits. if only one party member can go through walls then it's less useful because they get stuck by themselves with an enemy and no way to quickly get help from their buddies.
Another example of a way to conveniently walk through walls is to be an Earth Elemental (popular Druid form). They have a burrow speed and Earth Glide.

I also don't really agree with your "get stuck" point. Indeed you don't seem to have that issue with a Monk running behind enemy lines to take a crack at some squishies, so why would you have it with anyone else? Especially if they can easily get out, too? Or use it to kite? Or circumvent lockdown effects where the enemy absolutely isn't going to wait for the rest of the party to get out?


Personally I don't know why 500-600 feet seems to be your benchmark move speed. If one party member races off to go fight something that far away, it's going to take a while for their buddies to catch up and lend them a hand. You seem to be assuming that the only use for mobility is to run ahead of the party to engage a target in melee. Even if that were true (ignoring stuff like kiting, circumventing obstacles and hazards, powering through control spells like Plant Growth and Spirit Guardians and Eldritch Blast invocation pushes/speed debuffs, scouting, etc etc) and that was all you were using it for for some reason, it's not as hard to engage at those ranges as you seem to think. Longbows have a 600 foot range and ignore distance penalties with the ever-popular Sharpshooter. Sniper warlocks are also popular and effective (and can get really mobile to boot). Heck, a Wizard of level 5+ can keep their whole party on Phantom Steeds while traveling and thus can open a combat with an engagement range of at least 200 feet/round + ranged attack distance without spending any resources or even losing any overland travel time (since you can cast rituals while traveling). A Pegasus (for those with Find Greater Steed) moves at least 180 feet/round, and more if any mobility spells are shared to it (potentially a lot more).

On top of all of that, note that if an enemy has a superior engagement range, they aren't going to wait for the whole party to get to them. And while you may not have any sniper fights in your games, they clearly happen in some people's games because we occasionally get threads on here with people asking how to stand a chance in them after they fight one.

But mostly just because... you can totally do it without all that much investment, and the goalpost that Monks in general were claimed to reach was best mobility. Not "good mobility," best mobility. And 500 is better than 60, or 60+dash, or whatever.


All I really need for mobility is the ability to avoid AoOs reliably and a decent move speed, 100 feet/round is the highest you're ever likely to need imo, and it's trivial to build a monk capable of that.

Of course. But... you can totally do it with other classes too. The claim that is being disagreed with is that Monks have the best mobility in the game. If you say they're the best in the game at something, that doesn't merely imply that they can do a useful and effective thing, it implies that no others can do the thing on a comparable or superior level.

The point is that circumventing OAs and moving 100 feet a round is not actually that high a benchmark to reach. Doubly so if we're talking about doing it in a Monk-like fashion (such as spending bonus action / ki for Step of the Wind, at the cost of Flurry of Blows etc. Oh, and the Monk can dash OR disengage with Step of the Wind, not both).

Though, I'd disagree that 100 ft/round is all the mobility that'll ever matter even on battle maps of a more limited size. There are a lot of complications that can trip you up even with that speed, such as various control or kiting abilities. Plant Growth for example will cut your speed to 1/4th. Wall spells can just separate you if you can't get through them. Fliers are out of punching range regardless of your land speed. And so on.

stoutstien
2018-11-26, 01:53 PM
I do believe the design team put too much focus on one ability with the monk.
A Paladin loses smite and it's still a great class.
Druids lose wildshapes and still have full casting.
Monks lose SS and suddenly it's in a world of hurt.

KorvinStarmast
2018-11-26, 02:04 PM
Monks feel pretty good, but I have not played Kensei.
Shadow and Open Hand: good fun, enjoyable.
Four Elements: I think WoTC blew it with not making all elemental disciplines available. Just adding the flexibility would solve the FE perception of weakness; still limited by ki points, and the elemental powers aren't that powerful. But each seems to me to have a use.
The name of the class is "Way of the Four Elements" not "an element, and maybe another one."
:smallyuk:

Astofel
2018-11-26, 06:39 PM
snip

Well I asked to be schooled and you delivered, kudos friend. I guess I have no choice but to concede the point here. If you'll allow me to dive back into personal opinion, though, I'll say that the mobility the monk has feels better than other classes I've played. I think because they get to do all sorts of wacky stuff without having to cast spells, change shape, or explicitly use any magic monks feel amazing to play to me. It probably helps that I'm a martial artist myself so the fantasy of playing a supernaturally skilled one is very appealing to me.

Citan
2018-11-26, 07:44 PM
Honestly, I don't care so much about effectiveness as I care about the fantasy it's trying to sell and to me, that fantasy and the inspiration (and probably everyone else who see's this subclass) is pretty much Avatar. And that's where this subclass fails because you know so few techniques and can only use them so few times. A 4 Elements monk doesn't feel like a master of the Elements or even one Element. Like you can't even properly built around a single element. 4 Elements Monk for me doesn't suck because it's weaker than the other Monk subclasses, but because it fails to deliver what I want from it. To be fair, the Monk is probably a poor Chassis for a Avatar style bender of the Elements but that's an entirely different issue.
That I have no quarrel with. ^^ Even if I don't actually know well Avatar, I agree it would be hard to say you "master" 4 elements -or even one- when such mastery could only be expressed in so few ways.
For such a display, a mix between Land Druid or Wizard and Fighter or Rogue would be much more appropriate. :)


4e monk does feel way too limited compared to just grabbing a level or three of sorceror or wizard or druid or cleric or warlock or bard or pally or fighter or nearly anything with actual magic/combat.

I'd give them a "you get an extra Elemental Discipline at lvl3, because you don't have enough of them". And a "you get 1/2 your Monk level worth of Ki points *per day* to utilize in non-4e Ki uses, as a side Ki-pool, so you can still do normal Monk stuff. Ask your DM whether you round up or down". BTW, it was a typo, so "every Elemental Discipline is actually *4 levels lower on its prerequisites down to a minimum of lvl1*. You can also *expend one extra ki point on them than listed on the 4e-Monk-level/maximum-Ki-use chart*. Sorry, our editor forgot how to math and how d&d works", because we don't care if you don't have enough Ki for them yet or to use them regularly, but you can just do you and choose what you want at an appropriate level for those kinds of abilities.

That'd sort the problem. Would it make them powerful? Kinda. But honestly not that good anyway.
So basically you're saying be a Monk with on top of that the instant spellpower of a Warlock and the power flexibility of a regular caster? That would make them not "powerful", but extremely powerful.
You don't seem to realize how good Monks in general are, and 4e in particular compared to *martials*. Which brings me to the next point.


Hours of wall walking, free HP, blindsight, a ranged restrain (OK, crappy DC, but action-use removal). At level 2.

Also, it's not the only thing they do by level 9. By then they have uber summons, talking intelligent pets, massive f'you tank heals, big DoT spells, on-call flying (that can carry others), perhaps some stupid wildshape forms/always accessible abilities/wazoo spell-lists, and kinda free rein on how to approach matters.

It's not Druid > Monk, they're very different. But saying "I can wall-walk for cheap and punch people into comprising positions by lvl9" is not a good yardstick of what other classes can do at a similar level. Druids aren't actually that powerful either.
So you're comparing a martial and a fullcaster.
Sorry, I have no other word for it: it's far beyond illogical and void of any meaning.
Of course Druids can do hundreds of things more than Monk -or any martial-. But that mostly requires spell slots for that. So of course if you make 5mn adventuring days martials feel completely powerless in comparison. And they same would hold equally true with any pair of comparison martial / caster (yeah, Rangers and Paladins included).

Now, take a proper campaign with a mix of "fast" days and "enduring" days, you'll see how casters can feel powerless by the end while martials are still running up and high.

Also, you're further displaying your lack of hindsight with the last bit: Druids are the most powerful class taken in isolation, precisely thanks to (or because of) Wild Shape primarily, and the "prepared" varied spell list secondly.
Of course, with the right DM, Bards (breaking everything with Expertise), Sorcerer (manipulating whole world with Subtle), Wizard (filling head and book with all spells in the world) and Cleric (being "in bed" with his/her divinity so using Divine Intervention for crazy things beyond "replicate Cleric spell") can be much better.
But Druid is by far the most self-sufficient.



I've seen DMs really struggle in parties with Monks, and many a DM bemoan Stunning fist as being broken after seeing a Monk shut down a BBEG or scary CR'd monster by simply walking over and spamming Stunning fist on it, totally ending the encounter before it begins and stun-locking a priority target for a few rounds.

Monks are the most mobile class in the game, come with the best defences of any class in the game (eventually having all Save proficiencies, plus the ability to spend a Ki point to re-roll a save, plus a host of immunities) have good DPR (not Paladin or GWM Barbarian good, but more than adequate for frontline duties) and have the best at will Save or Suck class feature in the game (stunning fist) that is great for burning Legendary resistances (no monster wants to be stunned for a whole round) and for locking down priority targets.

They also come with great utility out of the box.

Monks can boost DPR simply by picking up a magic weapon just like any other class (a flame tounge short sword is a nice way to go as are any of the big ticket magical short swords, as is a Staff of striking), Bracers of defence work wonders on them also.

The only bad thing about Monks is once you figure out how to best use one as a player (identify target and isolate the nastiest creature in the encounter with Stunning fist, stun lock it and then wait for the Cavalry to arrive, and dont waste your time dealing with mooks) they become kind of repetitive:

Well, it's not a fault in class, it's a fault in player.
Shadow: sometimes keeping concentration on Darkness may be the best option to reduce enemy group's threat, while going up for a Stun would put it at great risk of breaking.
Long Death: instead of trying to stun that big oaf that probably has great Constitution, while not keep Ki and instead face him directly, soaking damage with THP while it lasts then using Dodge to waste a few more attacks? Or rushing before allies in front line to unleash a Fear before they shoot arrows?
Sun Soul/4E: soften a group of enemies with AOE, knowing that it allows allies to probably wipe out half of them in the remaining of the round?
Kensei: conserve ki and instead buff your longbow, because you expect Stun to be hard to land, whereas distance allows a free round of shots anyways?

Not every party is balanced.
Not every encounter will be followed by a rest.
Not every encounter will flow as expected.

If a player can do the same tactic every fight, then the problem lies with DM.



I do believe the design team put too much focus on one ability with the monk.
A Paladin loses smite and it's still a great class.
Druids lose wildshapes and still have full casting.
Monks lose SS and suddenly it's in a world of hurt.
Here too, I disagree.
Yeah SS is great, but it's not the always-win button Malifice describes.
Sometimes your party is in no shape to follow-up your stun.
Sometimes it's simply too dangerous to reach the VIT, or chance to land the Stunning Strike is too low.

So let's try and see what's left without Stunning Strike for each archetype, step by step.

Open Hand:
03. limited but decent -and cheap!- control at low level that stays relevant all game,
06. more resilience,
11. utility/fight opener buff,
17. top three feature of game.

Sun Soul
03. limited but decent ranged attack that scales with level (and boost for a ki),
06. very good action-economy short range AOE,
11. decent cost to benefit AOE with good damage type,
17. utility (from my understanding would repel Darkness) feature with mediocre damage.

Shadow:
03. great spells that stay relevant all game,
06. movement feature that is situational but stays relevant all game,
11. utility feature that stays relevant until high level (when there is too much detect magic and see invisibility around),
17. feature that expand occasions to try a Stunning Strike so becomes just an "expanded OA".

Kensei:
03. great weapon expansion that stays relevant all game + AC bonus that stays relevant all game (even if at highest levels it won't change much),
06. great ability that stays relevant all game (magic weapon + extra damage),
11. great ability that stays relevant all game (extra accuracy and damage for little cost),
17. good ability that stays relevant in all game even if without Stunning Strike it's just a decent accuracy boost.

Drunken Master:
03. great ability that stays relevant all game,
06. hard to pull/situational but extremely powerful feature (actually one of the top 10 class feature when your party builds around considering it scales so well),
11. ability that stays relevant all game (enemies should try more and more to impose disadvantage),
17. situational feature -decent but imo comes too late to be more than icing fluff on the Monk cake, especially with Stunning Strike out-.

Long Death:
03. great ability that stays relevant all game,
06. great ability that stays relevant up until last tier of play,
11. great ability that stays relevant all game,
17. decent ability that can be situationally good.

4E
03. access to
- "minimum damage that can be scaled + push/pull control",
- "free Disengage + fire damage + 10 feet for a ki",
- "AOE with chance to disengage"
- "AOE with decent damage for cost"
06. access to
- "situational but better ability than SS care arising (better condition, less resisted attribute)"
- "AOE upgrade that doubles as utility"
11. access to
- "be the best of Monk with 3d 60 feet speed upgrade"
- "AOE upgrade to get the best cost to benefit ratio instant spell"
- "situational but great case arising utility"
17. access to
- "best cost/benefit ratio AOE-timed spell"
- "great utility spell that can be used in various ways".

Yeah, not having Stunning Strike would certainly suck, but barring Kensei for which it's the only really defining feature apart from "just hit people", and possibly Sun Soul for which abilities don't scale well enough towards the end to my taste, I'd have no trouble having fun and being efficient with any archetype.

djreynolds
2018-11-26, 07:56 PM
As always Citan has a good post. The monk chassis is great

Grab the mobile feat at 4th, it saves KI

And get a bow, that's your ranger attack

stoutstien
2018-11-26, 07:59 PM
That I have no quarrel with. ^^ Even if I don't actually know well Avatar, I agree it would be hard to say you "master" 4 elements -or even one- when such mastery could only be expressed in so few ways.
For such a display, a mix between Land Druid or Wizard and Fighter or Rogue would be much more appropriate. :)


So basically you're saying be a Monk with on top of that the instant spellpower of a Warlock and the power flexibility of a regular caster? That would make them not "powerful", but extremely powerful.
You don't seem to realize how good Monks in general are, and 4e in particular compared to *martials*. Which brings me to the next point.


So you're comparing a martial and a fullcaster.
Sorry, I have no other word for it: it's far beyond illogical and void of any meaning.
Of course Druids can do hundreds of things more than Monk -or any martial-. But that mostly requires spell slots for that. So of course if you make 5mn adventuring days martials feel completely powerless in comparison. And they same would hold equally true with any pair of comparison martial / caster (yeah, Rangers and Paladins included).

Now, take a proper campaign with a mix of "fast" days and "enduring" days, you'll see how casters can feel powerless by the end while martials are still running up and high.

Also, you're further displaying your lack of hindsight with the last bit: Druids are the most powerful class taken in isolation, precisely thanks to (or because of) Wild Shape primarily, and the "prepared" varied spell list secondly.
Of course, with the right DM, Bards (breaking everything with Expertise), Sorcerer (manipulating whole world with Subtle), Wizard (filling head and book with all spells in the world) and Cleric (being "in bed" with his/her divinity so using Divine Intervention for crazy things beyond "replicate Cleric spell") can be much better.
But Druid is by far the most self-sufficient.


Well, it's not a fault in class, it's a fault in player.
Shadow: sometimes keeping concentration on Darkness may be the best option to reduce enemy group's threat, while going up for a Stun would put it at great risk of breaking.
Long Death: instead of trying to stun that big oaf that probably has great Constitution, while not keep Ki and instead face him directly, soaking damage with THP while it lasts then using Dodge to waste a few more attacks? Or rushing before allies in front line to unleash a Fear before they shoot arrows?
Sun Soul/4E: soften a group of enemies with AOE, knowing that it allows allies to probably wipe out half of them in the remaining of the round?
Kensei: conserve ki and instead buff your longbow, because you expect Stun to be hard to land, whereas distance allows a free round of shots anyways?

Not every party is balanced.
Not every encounter will be followed by a rest.
Not every encounter will flow as expected.

If a player can do the same tactic every fight, then the problem lies with DM.



Here too, I disagree.
Yeah SS is great, but it's not the always-win button Malifice describes.
Sometimes your party is in no shape to follow-up your stun.
Sometimes it's simply too dangerous to reach the VIT, or chance to land the Stunning Strike is too low.

So let's try and see what's left without Stunning Strike for each archetype, step by step.

Open Hand:
03. limited but decent -and cheap!- control at low level that stays relevant all game,
06. more resilience,
11. utility/fight opener buff,
17. top three feature of game.

Sun Soul
03. limited but decent ranged attack that scales with level (and boost for a ki),
06. very good action-economy short range AOE,
11. decent cost to benefit AOE with good damage type,
17. utility (from my understanding would repel Darkness) feature with mediocre damage.

Shadow:
03. great spells that stay relevant all game,
06. movement feature that is situational but stays relevant all game,
11. utility feature that stays relevant until high level (when there is too much detect magic and see invisibility around),
17. feature that expand occasions to try a Stunning Strike so becomes just an "expanded OA".

Kensei:
03. great weapon expansion that stays relevant all game + AC bonus that stays relevant all game (even if at highest levels it won't change much),
06. great ability that stays relevant all game (magic weapon + extra damage),
11. great ability that stays relevant all game (extra accuracy and damage for little cost),
17. good ability that stays relevant in all game even if without Stunning Strike it's just a decent accuracy boost.

Drunken Master:
03. great ability that stays relevant all game,
06. hard to pull/situational but extremely powerful feature (actually one of the top 10 class feature when your party builds around considering it scales so well),
11. ability that stays relevant all game (enemies should try more and more to impose disadvantage),
17. situational feature -decent but imo comes too late to be more than icing fluff on the Monk cake, especially with Stunning Strike out-.

Long Death:
03. great ability that stays relevant all game,
06. great ability that stays relevant up until last tier of play,
11. great ability that stays relevant all game,
17. decent ability that can be situationally good.

4E
03. access to
- "minimum damage that can be scaled + push/pull control",
- "free Disengage + fire damage + 10 feet for a ki",
- "AOE with chance to disengage"
- "AOE with decent damage for cost"
06. access to
- "situational but better ability than SS care arising (better condition, less resisted attribute)"
- "AOE upgrade that doubles as utility"
11. access to
- "be the best of Monk with 3d 60 feet speed upgrade"
- "AOE upgrade to get the best cost to benefit ratio instant spell"
- "situational but great case arising utility"
17. access to
- "best cost/benefit ratio AOE-timed spell"
- "great utility spell that can be used in various ways".

Yeah, not having Stunning Strike would certainly suck, but barring Kensei for which it's the only really defining feature apart from "just hit people", and possibly Sun Soul for which abilities don't scale well enough towards the end to my taste, I'd have no trouble having fun and being efficient with any archetype.

i'm not saying that monks are useless without SS but it hard to argue that it's not the single most power offensive feature in the class (maybe in the game) and could be the cheapest way to burn legendary resistance. monks are nice due to pretty much being great at being ok at a lot of different things but without SS they would be a minor nuisance to higher CR foes.

LudicSavant
2018-11-26, 08:49 PM
Well I asked to be schooled and you delivered, kudos friend.

NP! :smallsmile:

Malifice
2018-11-26, 10:43 PM
If losing half your attacks (by foregoing Flurry of Blows for Step of the Wind or the like) doesn't count as losing some of your offense, I'm not sure what does.

Do they though?

Monks gain an incremental increase to speed +10' at level 2, +15' at level 6, +20' at level 10, +25' at level 14 and +30' at level 18.

At 9th level they can use any of their movement to walk along liquids or vertical surfaces. Lakes, rivers, or even walls lower than 150' simply dont bother them anymore. A 10th level Monk can simply Dash [action] Step of the Wind [bonus action] to gain a movement of 150' for the round, walking up any walls or across any water without breaking stride.

Unlike with the Rogue, the monks extra speed doesnt require a bonus action to use. It's simply always on. They also gain Step of the Wind (bonus action disengage or dash, plus double jump distance) which is analogous to Cunning action if they need even more speed.

And you cant say that Rogues dont also lose offence by using Cunning action to Disengage and Dash (instead of Hide for advantage or fight with the off hand).

An 18th level Monk can simply use his movement to move 60' while retaining his bonus action to Flurry or whatever. He can bonus action Step of the wind to move 120' or go 'all out' with movement of 180' if he really want to. Your Rogue of the same level moves 30' unless he uses his Bonus action to move 60'.

Slap on Astral Projection at 18th level and various Archetype abilities and the Monk has a level of manouverability and mobility that is damn hard to beat.

djreynolds
2018-11-26, 10:58 PM
It really is great class.

And in a party very "cheap to feed" in regards to armor and weapons

My advice, humbly so, grab the mobile ASAP, it saves on KI and will keep you alive by getting you out of melee. Now you can FOB and stun and leave it for the fighter to clean up.

And of course, just grab a short bow or long bow, too easy.

The way of the 4 elements isn't terrible, you're still a monk, but the ability to cast hold person versus wisdom may prove easier than stunning fist and con saves.

Remember legs and heabutts are unarmed strike, so use a staff or spear two-handed for you main attack and then kick with you BA.

Your higher wisdom stat will make you great with insight, grab a background that gives you access to perception if you want.

Good luck

LudicSavant
2018-11-26, 11:18 PM
Do they though? Yes. If they use any of their bonus action mobility.

If you don't want to count any of your bonus action mobility, then no, you aren't cutting into your offense. But that just makes the benchmark we already beat without cutting into our offense lower, so I'm not sure what your point is.

Malifice
2018-11-27, 01:25 AM
Yes. If they use any of their bonus action mobility.

If you don't want to count any of your bonus action mobility, then no, you aren't cutting into your offense. But that just makes the benchmark we already beat without cutting into our offense lower, so I'm not sure what your point is.

Monks get a bonus to movement speed greater than any other class, plus the ability to use that movement to walk up walls and over liquids. At will with no resource expenditure.

Thats before we factor in actions or bonus actions to increase that movement and mobility further, or any sacrifice in offence at all.

Yeah if you desperately need to move 180' in a round using your actions you can. Best a Rogue can do in the same situation is move 90' using all his actions for movement.

Monks are incredibly mobile. Any other class that wants to mirror that mobility also needs to expend an action or resource like a spell slot, so I'm a little confused as to what your argument is here.

LudicSavant
2018-11-27, 02:46 AM
Monks get a bonus to movement speed greater than any other class, plus the ability to use that movement to walk up walls and over liquids. At will with no resource expenditure.

Thats before we factor in actions or bonus actions to increase that movement and mobility further, or any sacrifice in offence at all.

Yeah if you desperately need to move 180' in a round using your actions you can. Best a Rogue can do in the same situation is move 90' using all his actions for movement.

Monks are incredibly mobile. Any other class that wants to mirror that mobility also needs to expend an action or resource like a spell slot, so I'm a little confused as to what your argument is here.

Uhm? None of the examples I gave of outmaneuvering Monks were Rogues. Nor was there some arbitrary “no using any resources” goalpost in place (indeed, I assumed the Monk was using theirs), though you can in fact beat a 60 foot move speed without using an in-combat Action or one of the day’s spell slots.

+30 feet of movement at level 18 just isn’t enough to make you “the most mobile” in D&D.

Dark Schneider
2018-11-27, 02:46 AM
2. There are no magical fist weapons.
I understand that the people at WOTC do not really care about martial gear or equipment it is just an afterthought but the idea that there are no magical gloves/gauntlets/whatever is just bad design and insulting. Essentially, don't bother playing a monk in a by the book game or AL, when everyone else has magical +1 or 2 gear, you will get nothing. If you do get a magical monk weapon, it only covers half your attacks in a round. You still have to attack unarmed with your bonus actions.

3. They need so many stats that you will not really get to customize much with feats unless you are a variant human. Even then, there are not that many great monk feats.
2. If we look at DMG, it says "Weapon +1, +2, +3". You could perfectly put gloves as weapon, I think no DM will be reticent to this. For armors, monks can wear tunics. If defense worries you more than mobility, you could get Light Armored feat, maximize DEX, and leave WIS low. But WIS low only fits for Shadow Monk IMO, that has no saving throw abilities.

3. Monk really need to focus on DEX, grants AC, skills, initiative, attack and damage. If you have a monk that uses abilities with saving throws, then also WIS, but no need to maximize. You could decide to maximize WIS and leave DEX at 16 (+3), depending how you plan to use the monk.

Monks can also be good rogues, less skills, better in combat (multiple attacks). The Shadow Monk, with some background granting thieves tools proficiency, and focused on Deception, Stealth, Perception, and skills like those, with Skilled Feat to complete the set of skills, can give a good "martial rogue" or rogue-monk. Using WIS to add AC you have an AC similar to rogue, also have Evasion ability, with Ki you can also Evade or Disengage as bonus action, and have even more mobility and better in combat.

And, we have many useful feats for monks.
- Actor: if you get proficiency with disguise tools, you have an infiltrator.
- Alert: always useful.
- Dungeon Delver: for rogue-monks.
- Grappler: monks are great grapplers, as they have always hands free.
- Lucky: always useful.
- Mage Slayer: with your movement speed, put close to foe caster.
- Martial Adept: get some combat maneuvers.
- Mobile: of course.
- Skilled: for rogue-monks.

A Shadow Monk made as rogue-monk AKA Ninja is one of the most funny characters to play, and so versatile.

Malifice
2018-11-27, 03:47 AM
Uhm? None of the examples I gave of outmaneuvering Monks were Rogues. Nor was there some arbitrary “no using any resources” goalpost in place (indeed, I assumed the Monk was using theirs), though you can in fact beat a 60 foot move speed without using an in-combat Action or one of the day’s spell slots.

+30 feet of movement at level 18 just isn’t enough to make you “the most mobile” in D&D.

Without using a spell slot, which base class is faster and more mobile than the Monk?

LudicSavant
2018-11-27, 05:22 AM
Monks get a bonus to movement speed greater than any other class, plus the ability to use that movement to walk up walls and over liquids. At will with no resource expenditure.

Thats before we factor in actions or bonus actions to increase that movement and mobility further, or any sacrifice in offence at all.



you can in fact beat a 60 foot move speed without using an in-combat Action or one of the day’s spell slots.

+30 feet of movement at level 18 just isn’t enough to make you “the most mobile” in D&D.

Without using a spell slot, which base class is faster and more mobile than the Monk?

So, let's make the parameters clear here so that the goalposts don't get moved again after I answer. You are saying that the +30 foot movement speed on its own, without factoring in anything that sacrifices your offense or burns ki, is superior to any other class, yes?

And if I show you a way that I can beat this +30 foot movement + ability to run up walls all day, without spending a spell slot or using up in-combat bonus actions or actions of my own, you will concede the (new) point, yes?

SleepIncarnate
2018-11-27, 05:28 AM
Are you going to use the example of a paladin on a pegasus/griffon mount? Cause we're getting off the topic here too.

LudicSavant
2018-11-27, 05:35 AM
Are you going to use the example of a paladin on a pegasus/griffon mount?

I actually had several different examples in mind. Though that one would certainly work, if he doesn't count your companion that you summoned 3 adventures ago as "using a spell slot." But I was going to pick something different just to sidestep him potentially saying that that doesn't count.

Malifice
2018-11-27, 07:43 AM
I actually had several different examples in mind. Though that one would certainly work, if he doesn't count your companion that you summoned 3 adventures ago as "using a spell slot." But I was going to pick something different just to sidestep him potentially saying that that doesn't count.

Pick any class that is more mobile and maneuverable than the monk without using spells.

Wild shaped druid perhaps. But (barring moon druid) you're also sacrificing offence in wild shaping seeing as it shuts down all your other class features (spells) while wild shaped

LudicSavant
2018-11-27, 11:01 AM
Pick any class that is more mobile and maneuverable than the monk without using spells.

Wild shaped druid perhaps. But (barring moon druid) you're also sacrificing offence in wild shaping seeing as it shuts down all your other class features (spells) while wild shaped

*Points to post #70, waits patiently.*

Malifice
2018-11-27, 12:33 PM
*Points to post #70, waits patiently.*

I asked a specific question. How much more specific do you want?

stoutstien
2018-11-27, 12:49 PM
Question. How often does a player need 90+ feet of movement in a round? And if you are that much faster than everyone aren't you risking being isolated.

Jamesps
2018-11-27, 01:27 PM
Question. How often does a player need 90+ feet of movement in a round? And if you are that much faster than everyone aren't you risking being isolated.

Depends on how creative the situation allows you to be.

Keep in mind that while grappling your movement is halved. Also while encumbered I believe.

I played a highly mobile character and I would often use my mobility to transport allies and enemies around the battlefield. Also, keep in mind that skirmishing tactics often require you to end your turn out of sight or under cover or concealment. This means you effectively need double the movement of anyone else if you want to rush out, hit them, then rush back.

LudicSavant
2018-11-27, 01:39 PM
I asked a specific question. How much more specific do you want?

Specific enough to answer the two clarifying, yes or no questions in post #70, obviously.

KorvinStarmast
2018-11-27, 02:35 PM
Without using a spell slot, which base class is faster and more mobile than the Monk? Or jumps higher. Step of the Wind seems to have a feature I have never exploited properly.

I've been reading up posts by a number of folks who feel that the doubled jump distance for step of the wind also doubles the height of the high jump (running or standing).
How have you seen it ruled at your tables?
The way I initially read it, the jumping distance text seemed to imply only the running long jump (increased distance) while not mentioning height ... polling DM's on this to see what the gut feel is. (If we give the monk ogre gaunts ... str 19, bonus +4, running high jump gets to 7 feet ... double to 14 ...)


Step of the Wind

You can spend 1 ki point to take the Disengage or Dash action as a bonus action on your turn, and your jump distance is doubled for the turn.

LudicSavant
2018-11-27, 03:23 PM
Or jumps higher. Step of the Wind seems to have a feature I have never exploited properly.

I've been reading up posts by a number of folks who feel that the doubled jump distance for step of the wind also doubles the height of the high jump (running or standing).
How have you seen it ruled at your tables?
The way I initially read it, the jumping distance text seemed to imply only the running long jump (increased distance) while not mentioning height ... polling DM's on this to see what the gut feel is. (If we give the monk ogre gaunts ... str 19, bonus +4, running high jump gets to 7 feet ... double to 14 ...)

JUMPING
Your Strength determines how far you can jump.
Long Jump. When you make a long jump. you cover a number of feet up to your Strength score if you move at leasl 10 feet on foot immediately before the jump. When you make a standing long jump, you can leap only half that distance. Either way, each foot you clear on the jump costs a foot of movement.

This rule assumes that the height of your jump doesn't matter, such as a jump across a stream or chasm. At your DM's option, you must succeed on aDC 10 Strength (Athletics) check to clear a low obstacle (no taller than a quarter of the jump's distance), such as a hedge or low wall. Otherwise. you hit it. When you land in difficult terrain. you must succeed on a DC 10 Dexterity (Acrobatics) check to land on your feet. Otherwise. you land prone.

High Jump. When you make a high jump you leap into the air a number of feet equal to 3 + your Strength modifier if you move at least 10 feet on foot immediately before the jump. When you make a standing high jump, you can jump only half that distance. Either way, each foot you clear on the jump costs a foot of movement. In some circumstances. your DM might allow you to make a Strength (Athletics) check to jump higher than you normally can.

You can extend your arms half your height above yourself during the jump. Thus. you can reach above you a distance equal to the height of the jump plus 1.5 times your height.


I'd rule that you get to jump twice as high, as height is just vertical distance (eats into your move speed and everything).

Though, since you're probably not investing in Strength as most Monks, your doubled jump height tends to just be rather unimpressive (example: 8 strength doubled high jump is 4 feet, whereas a 20 strength character's non-doubled high jump in plate armor is 8 feet).

KorvinStarmast
2018-11-27, 03:38 PM
I'd rule that you get to jump twice as high, as height is just vertical distance (eats into your move speed and everything). OK. I am very familiar with the relevant rules, having addressed this point here (https://rpg.stackexchange.com/a/136249/22566). I am interested in how people have ruled on the SoTW thing, given how the words are laid out in the Step of the Wind text. Thanks for your take on that. :smallsmile:


Though, since you're probably not investing in Strength as most Monks, your doubled jump height tends to just be rather unimpressive (example: 8 strength doubled high jump is 4 feet, whereas a 20 strength character's non-doubled high jump in plate armor is 8 feet). Yes. Hence the mention I made of a monk with Ogre Gaunts.

LudicSavant
2018-11-27, 03:44 PM
Yes. Hence the mention I made of a monk with Ogre Gaunts.

*Nod*

Yeah, I'd totally give you the 14 foot jump height. Seems to me that this is just the RAW (vertical distance is still distance. And even if it wasn't, the wording in Long Jump for the height option gives you height based on your horizontal distance).

KorvinStarmast
2018-11-27, 04:01 PM
*Nod*

Yeah, I'd totally give you the 14 foot jump height. Seems to me that this is just the RAW (vertical distance is still distance. And even if it wasn't, the wording in Long Jump for the height option gives you height based on your horizontal distance). Yeah. The long jump is a function of strength score, the high jump a function of strength bonus. I can see how that ruling makes sense.

PandaPhobia
2018-11-27, 04:42 PM
Hi yall

So I've never played a monk. Certainly wouldn't mind try it (acutally considered it as my first melee class since my two other chracters thus far are somewhat ranged) but until then...
Looking at their skill lines and perks, they seem pretty self sufficient and functional. But that's usually where it stops. Most people, if i glance, will say that monks are 'ok'. So that doesn't tell me if people are saying they're happy or they wish there was more.

So to the people who play them: Whats it like playing a monk.
Is there anything you would change? Do you feel good playing it? Are there things you wish you had or things you wish got a little bump?

I'm reminded when people complain about the ranger (that they're functional but are a bit of a mess) and sorcerors (who are functional but require some of spell planning to make work properly)
Is this the same kinda issue or am making an assumption

monks are awesome for zipping around the battle field, especially when paired with a mobile feat. the thing about monks is that they can take on a lot of roles in the party, with dex being so important in 5e. they can be stealthy rogue replacements, tanky fighter replacements, or even blasty sorcerer replacements. the thing they are best at however is messing with a lot of enemies at once from the frontlines. for that I recommend a kensai monk, b/c they get that sweet ac bonus whenever they attack, as well as what is essentially a magic weapon feature.

PandaPhobia
2018-11-27, 04:43 PM
Question. How often does a player need 90+ feet of movement in a round?

always. you always need 90 feet of movement, especially with a ranged kensai monk build.

KorvinStarmast
2018-11-27, 05:12 PM
always. you always need 90 feet of movement, especially with a ranged kensai monk build.
*golf clap* my monks all agree with you. :)

Citan
2018-11-27, 07:40 PM
So, let's make the parameters clear here so that the goalposts don't get moved again after I answer. You are saying that the +30 foot movement speed on its own, without factoring in anything that sacrifices your offense or burns ki, is superior to any other class, yes?

And if I show you a way that I can beat this +30 foot movement + ability to run up walls all day, without spending a spell slot or using up in-combat bonus actions or actions of my own, you will concede the (new) point, yes?


Uhm? None of the examples I gave of outmaneuvering Monks were Rogues. Nor was there some arbitrary “no using any resources” goalpost in place (indeed, I assumed the Monk was using theirs), though you can in fact beat a 60 foot move speed without using an in-combat Action or one of the day’s spell slots.

+30 feet of movement at level 18 just isn’t enough to make you “the most mobile” in D&D.
I'm genuinely interested in what you'll throw out here.
Disclaimer: don't know for Malifice, but for me, any magic item is out. Any concentration spell is out. Any effect that lasts less than 8 hour is out.
I guess everything else is fair game.
Only thing I see right now is really the Steed spells.

As for why I'm putting such restrictions, it's because as Malifice tried to stress out, Monk's mobility is *always* on. It may not make a big deal for 5-mn adventuring days, or in large parties where you can find someone to do the scouting job or that can buff you by spending many slots.

In harsher campaigns where resources are scarce, every martial shines greater because they are "always-on guys/gals" in essence.


Question. How often does a player need 90+ feet of movement in a round? And if you are that much faster than everyone aren't you risking being isolated.
Always?
Speed is power. Sadly not many people really grasp it around here (not targeting you ^^). XD

You seem to think of 90 feet as a straight move, but it's really the most plain (and not necessarily the brightest indeed) way to use such speed.
However, 90 feeet means...
1. Straight way and back of 45 feet which is wider than most creature's movement. Paired with Mobile, it simply means you have a "50 feet melee reach".
(or 60 if 4E and using Fangs of Fire Snake although I see no reason for that at that level XD)
(or 90 as a 4E with Fly).

2. Ability to close in, move up a tall wall (like 30 or 40 feet) and still have enough movement to attack a target then position self in a decent position (or even, you know, let yourself fall back down, using your reaction to make damage irrelevant).

3. Ability to reach a target in melee, that would be normally easily reachable with normal speed, but is guarded by a half-moon guards. Extra speed + jump + wall run means that contrarily to others, you have a way to reach without taking (m)any OA on the way (of course, it would be wise to refrain from trying that if you don't have at least 25 feet of movement left after that, unless you're very confident in avoiding a rain of sword attacks ^^).

4. Ability to position enemies: that one requires investment, either a STR Monk (strange, but doable in multiclass), or in Expertise one way or another to make it reliable, unless you're Open Hand / 4e. But Grappling can be an extremely potent way to build up damage by synergizing with other (pulling/pushing into an existing AOE, putting enemy in range of Sentinel ally's OA, closing creatures together for upcoming splash damage etc).

5. Ability to help anyone: should a backliner need immediate support because an enemy managed to sneak around and ambush, good luck getting Barbarian or Paladin. Best they can do is probably either Dash and make him flee or try thrown attacks with lesser efficiency (especially if enemy use your pal as cover). You on the other hand, can reach and try to disable in a single turn. This also makes a Monk a potent off-hand healer with Spare the Dying cantrip or Healer feat. :)

sophontteks
2018-11-27, 07:58 PM
Question. How often does a player need 90+ feet of movement in a round? And if you are that much faster than everyone aren't you risking being isolated.
Yep always. The battlefield is often based on the 30 foot/60 foot ranges. Either you can walk up and hit them, or you have to dash. A monk can walk and hit people in the 60 foot mark and dash hit pretty much anyone they want.

Ideallly a monk would hit an obscure target without provoking reactions and retain enough movement to kite the melee enemies afterwards. Its literally the opposite of your fears. The more movement a monk has, the better he is at avoiding being caught isolated while retaining his role as a flanker/single target CCer.

Remember that the monks kit is incredibly strong vs. ranged attacks, spells, and saves.

Angelalex242
2018-11-27, 10:03 PM
How does a monk feel? With his hands, presumably. ;)

Sindal
2018-11-28, 01:02 AM
How does a monk feel? With his hands, presumably. ;)

Eyyyyyy *fingerguns*

Well looking at the posts we've gotten. Monks seem pretty stable.
-High in flavour.
-Easy to build
-pretty fast
-adaptable to assault, pinner or supporting others aggresively
- pretty ok subclasses
-learning all languages is pretty cool

Some downsides include
- a leaning towards ranged damage problems if not catered for.
- a magical item tailoring issue
- made to be adaptable but not to shine

This was a very informative thread guys. I hadnt expected it to get much attention.

Dark Schneider
2018-11-28, 04:58 AM
- made to be adaptable but not to shine

The best description.

Monks can handle many things in a good way, but rarely will be the best one in anything.

But how many things can acomplish well with a single character, I think few can do that. The same monk can:
- Do rogue work: by background for proficiencies, and getting with background and race proficiency on stealth, and something like those.
- Do ranger work: with Skilled feat, get Survival and some others to complete the skills set. Using short bow with multi-attack (that the rogue does not have).
- Do fighter work: has good abilites for melee combat (including stunning) and many multi-attack.
- Do wizard work: with some kind of movement spells applied to self, and in the case of Four Elements Monk can begin the combat with the AoE spell to deplete some foes HPs. Also understand all languages.
- Do cleric work: as WIS is one of its main stats, it can easily multiclass with cleric at least a few levels, depending if you want all the monk abilities or not. IMO the lvl 20 is not needed at all, as with a short rest you recover all the ki. So 1 cleric lvl allows you to stand up knocked out allies (healing) and also use cleric spell scrolls (with a roll check, but can try at least), for revival/resurrection scrolls.

Depending the build, can achieve a good number of these. But, if you look for fighter replacement as the brute in combat, better go for a fighter.

But the monk is not necessarily the replacement, you can use it to help others. Help the rogue (giving advantage to picking locks or with traps as also have Evasion and most traps use DEX saves), help the fighter in combat in a very decent manner, help the group translating languages, help the group maneuvering (like jumping to activate something or going to the other side of the lake to bring the boat), etc. Can become the man for everything that can assist to many things.

Citan
2018-11-28, 06:28 AM
Eyyyyyy *fingerguns*

Well looking at the posts we've gotten. Monks seem pretty stable.
-High in flavour.
-Easy to build
-pretty fast
-adaptable to assault, pinner or supporting others aggresively
- pretty ok subclasses
-learning all languages is pretty cool

Some downsides include
- a leaning towards ranged damage problems if not catered for.
- a magical item tailoring issue
- made to be adaptable but not to shine

This was a very informative thread guys. I hadnt expected it to get much attention.

I think this is a pretty good summary, although some people here (including me ^^) would stay that anytime you make a decisive, winning-turn action like Stunning a dangerous opponent or wasting a caster's concentration on nasty spell is shining enough of a moment. :)

But overall, yeah, Monk is very efficient, in a very non-flashy way. :)

mephnick
2018-11-28, 10:33 AM
I have problems with my main ability being reliant on DCs considering how many DMs have come out as Pro-Fudge lately.

Misterwhisper
2018-11-28, 11:13 AM
I have problems with my main ability being reliant on DCs considering how many DMs have come out as Pro-Fudge lately.

Yeah I have the same issue.

I am not basing my main class power on if the DM feels like letting me shut someone down or not.

I almost never use SS, I just tend to stick to more Flurry uses or others.

At least when the DM lies and says that the Wolf is still standing after taking 60ish HP it is obvious as compared to them saving 4 times in a row.

Deathtongue
2018-11-28, 11:24 AM
Wait, what? DMs fudging saves is a thing?

You absolutely cannot do that with a monk. Monks aren't a bad class, but Stunning Strike is their big saving grace. It's more important to them than smiting is to a paladin. If DMs are nerfing Stunning Strike, then they're seriously at the level of pre-Hexblade Bladelocks or unerrata'd BM Rangers.

Misterwhisper
2018-11-28, 11:34 AM
Wait, what? DMs fudging saves is a thing?

You absolutely cannot do that with a monk. Monks aren't a bad class, but Stunning Strike is their big saving grace. It's more important to them than smiting is to a paladin. If DMs are nerfing Stunning Strike, then they're seriously at the level of pre-Hexblade Bladelocks or unerrata'd BM Rangers.

I hope you are being sarcastic with the first line.

I have NEVER played with a DM that does not fudge dice, and pretty often.
Many do not even bother trying to hide it.

Ex.

Player: Ok I cast Hold Person on the BBEG.
DM: That stuns them and makes all hits cries on them right?
Player: Yeah, Save DC is 15.
DM: *does not even pick up dice* Yeah he makes that, I am not having my encounter trivialized.

Almost every DM I have played with runs a "Rail" game as I call it.

They have a story, usually a pretty good and compelling one.
However:
You will never get the better of certain plot important NPC's until you are supposed to.
Combats will last as long as the DM thinks they need to in order to make you waste resources.
You will never circumvent a situation that is important with the use of clever spells or skills.
No important NPC will ever fail a save or lose check.

That is why when I run a game I am sitting at the table with the player and do not have a DM screen.
I roll my dice in the open where everyone can see it and if you outsmart the situation with clever RP and things great, congratulation, but I will also not pull punches to save you if you do something stupid.

mephnick
2018-11-28, 11:43 AM
Wait, what? DMs fudging saves is a thing?

Anything that makes the session more fun!! (I get to decide what's fun because I'm the guy with the screen)

There is no good argument for fudging, but people sure try. Even guys like Matt Colville admit to doing it for the sake of the "story".

Deathtongue
2018-11-28, 11:50 AM
I hope you are being sarcastic with the first line.

I have NEVER played with a DM that does not fudge dice, and pretty often.
Many do not even bother trying to hide it.I'm not being sarcastic. I play a lot of home games, online games, and AL. This has not been my experience. I get annoyed at a lot of stuff DMs do, but fudging has not been one of them. Half of them make rolls out in the open and the other half I don't get that vibe. Yeah, there are some DMs that are on a hot streak but the way they allow certain anticlimaxes (i.e. an AL game I was at last Saturday a monk threw out 4 Stunning Strikes against an End-of-Session Lich, burned through their LSes, and nailed the last one on the first round -- and this is not a probable outcome!) makes it feel real.

That was specifically the example I had in mind when I say that monks have problems but they also have their moments. You can't nerf that. That's like nerfing the barbarian's ability to rage.

mephnick
2018-11-28, 12:01 PM
I played in a game where I threw 4 SS at an enemy caster, also causing 4 concentration checks on the spell that was making the fight difficult. My friend (DM) saved all 8.

Ok good luck, maybe..except I'd ran that module before and knew the caster had a +0 Con save and no legendary saves. I don't know what the odds of rolling 10+ and 15+ 4 times in a row is...but needless to say I got the feeling the DM didn't want the combat "ruined" by the monk actually cancelling the show-stopper spell. He admitted after the campaign that he fudged a few of them. These are the situations I roll in the open to avoid such second guessing. That single combat ruined that character for me.

Sahe
2018-11-28, 12:46 PM
On the danger of turning this into a discussion about fudging:

I am on the side of Matt Colville on this: the dice promises drama, but not always delivers. When you sit down with a player and make a character and figure out how their Backstory is interconnected to the BBEG, I want them to triumph or perish in the fight against that BBEG...not being randomly insta critted to death by Goblin number 4 because they were only level 1. There is nothing dramatic or fun about that. Especially if it's because I miscalculated an encounter. If you run a narrative game about heroes who save the world or some such, leaving everything up to the dice fudging in my opinion is necessary, within reason at least.

Another thing where I fudge often is when NPC are in play. NPCs aren't there to steal the player's thunder so they usually won't last hit important enemies, just deal enough to leave them at death's door.

Fudging dice is for narrative games where you want to play story. It doesn't belong in dungeon crawls and other games more focused on combat mechanics.

mephnick
2018-11-28, 02:39 PM
I want them to triumph or perish in the fight against that BBEG...not being randomly insta critted to death by Goblin number 4 because they were only level 1. There is nothing dramatic or fun about that..

...then don't run the goblin fight. If there's no consequence it shouldn't be in the game.

The dice should tell the story and the players and DM should react to the fiction they create. The character being killed by a goblin should become part of the world and the story that is told after the campaign. Have your characters react to it. That character wasn't tough enough to be an adventurer. What does that say about the world? What does it say about the decision to be an adventurer? What does it say about goblins? Give the surviving characters PTSD. Make that goblin a recurring enemy. What's special about him?

Fudging is 100% the DM deciding what is fun and enforcing that on the group. Who's to say that character dying at level 1 doesn't improve the story going forward? You? Why do you get to decide that?

KorvinStarmast
2018-11-28, 03:04 PM
Player: Ok I cast Hold Person on the BBEG.
DM: That stuns them and makes all hits cries on them right?
Player: Yeah, Save DC is 15.
DM: *does not even pick up dice* Yeah he makes that, I am not having my encounter trivialized. Wow, playing with jerks can't be a good experience. The bolded part only needs one response:
It's not all about you.

GlenSmash!
2018-11-28, 03:14 PM
Yikes.

When I DM I roll in the open so we can all celebrate or commiserate together.

stoutstien
2018-11-28, 03:41 PM
I only fudge rolls in parties favor never aginst them.

GlenSmash!
2018-11-28, 04:20 PM
I only fudge rolls in parties favor never aginst them.

This is also reasonable.

I mean if my players find a way to trivialize a combat good for them! They are heroes after all.

Sahe
2018-11-28, 06:12 PM
...then don't run the goblin fight. If there's no consequence it shouldn't be in the game.

The dice should tell the story and the players and DM should react to the fiction they create. The character being killed by a goblin should become part of the world and the story that is told after the campaign. Have your characters react to it. That character wasn't tough enough to be an adventurer. What does that say about the world? What does it say about the decision to be an adventurer? What does it say about goblins? Give the surviving characters PTSD. Make that goblin a recurring enemy. What's special about him?

Fudging is 100% the DM deciding what is fun and enforcing that on the group. Who's to say that character dying at level 1 doesn't improve the story going forward? You? Why do you get to decide that?

We seem to fundamentally disagree on this issue or run/play in fundamentally different games. I do challenge however the idea that the last instance at the table should be the chance and possibilities of a random die roll. In the end, yes I as a DM decide what is fun and I'm enforcing that on the group which sounds like an incredibly hostile term, but leaving it a 100% to the dice is in the end just shifting the responsibility of telling a story and drama to the dice...and they don't always deliver. After all, we're not playing a boardgame.

Citan
2018-11-28, 06:25 PM
Anything that makes the session more fun!! (I get to decide what's fun because I'm the guy with the screen)

There is no good argument for fudging, but people sure try. Even guys like Matt Colville admit to doing it for the sake of the "story".
There ARE good arguments for fudging, both ways even mind you (sometimes you find an idea very interesting, and players are all very much into it, so you give it an actual chance to work even though normally opponent's saves it would be plain impossible or at least extremely unlikely).

However, as many things in DM's responsability, such things should be very scarce and imo always towards a commonly defined "interest and fun" balance.

For example...


They have a story, usually a pretty good and compelling one.
However:
You will never get the better of certain plot important NPC's until you are supposed to.
Combats will last as long as the DM thinks they need to in order to make you waste resources.
You will never circumvent a situation that is important with the use of clever spells or skills.
No important NPC will ever fail a save or lose check.

That is why when I run a game I am sitting at the table with the player and do not have a DM screen.
I roll my dice in the open where everyone can see it and if you outsmart the situation with clever RP and things great, congratulation, but I will also not pull punches to save you if you do something stupid.
This is not something I'd advocate ever for DMs, neither would I appreciate as a player.
Doing this once is ok, telling your players at end of session "sorry guys, I'm still kinda inexperienced, and you came up with such a good tactic/amazing out-of-the-box I panicked. I'll strain myself and let it go next time."

Doing this as the "normal" way of DM-ing? Unless all players are fine with "playing a videogame japanese RPG with 5e class system", it's really not a good approach. :/

Your own story also, Mephnick, is definitely a very bad use of DM's prerogative, not only because he gives absolutely no chance for you to work your abilities -which are things he *should* have taken into account when designing an encounter-, but by presenting it as natural instead of just apologizing or better asking if you would be ok with it.


You can't nerf that. That's like nerfing the barbarian's ability to rage.
Nice summary of what was put on Mephnick. XD


...then don't run the goblin fight. If there's no consequence it shouldn't be in the game.

The dice should tell the story and the players and DM should react to the fiction they create. The character being killed by a goblin should become part of the world and the story that is told after the campaign. Have your characters react to it. That character wasn't tough enough to be an adventurer. What does that say about the world? What does it say about the decision to be an adventurer? What does it say about goblins? Give the surviving characters PTSD. Make that goblin a recurring enemy. What's special about him?

Fudging is 100% the DM deciding what is fun and enforcing that on the group. Who's to say that character dying at level 1 doesn't improve the story going forward? You? Why do you get to decide that?
NO.
There is no "should".
You lead games like this, and that's perfectly fine.
That's 95% the way I lead games too, and I'm fine with that.
But I'm fine with that because...
- I play with a very small group of players.
- We are all fine with story going in all directions at times (read: I improve 99% of what happens on the fly).
- I'm very bad at finding names but otherwise good enough at reacting/providing ideas on the fly. Usually.
- We all share (mostly) the same kind of humour and the same taste in adventures, so what I improvise usually fits with what players expect.
- And obviously we are playing in a custom setting, with very shallow storyline.

If any two of these statements were wrong, I would probably fudge rolls occasionally, simply because, as everyone agree here I think, DMING IS HARD.
Not everybody has the same skills nor the same experience with that, especially when dealing with impromptu things. Theorically, you should always have a proper answer to whatever idea players may come with. Theorically, you should consider each and every NPC you create as fodder that may die any moment at a player's whim.
But keeping true to those is not easy.
And there is often social pressure coming from players. Even if those didn't put any intent into it, and especially on people who are starting DMing.
Not every DM can simply forbid self to fudge ever and bravely face the risk of ending in a "blank page" state because players want to derail a published adventure, or seeing hours of work run down the toilet in minutes because there was something about encounter that was completely forgotten about. That "self-distance" comes mainly with more play unless your own real-life background already helped forge it.

As long as fudging is not his/her "usual way of rolling" and everyone had a chance to tell about what they expect of the game in session 0, I would have no quarrel with a DM fudging, if only because no fudge would completely break the tension, and thus, the immersion. And I would always prejudge that he/she did it in good faith and in the human group's interest. Unless there is really, really something extremely illogical happening as a result.
That's about trust relationship, or, more egoistically, "keeping your DM in shape of DMing". XD
There are not many DMs around as it is, so as a player one has to be tolerant. It's not incompatible with the fact that the way of running game may be a dealbreaker. :)

Sindal
2018-11-29, 01:03 AM
Looks like the topic of the thread changed.
I dont mind but shoulsnt you make a seperate thread for it?

Dark Schneider
2018-11-29, 04:36 AM
There are never good arguments for fudging. The role-playing is not like a book, nothing is written, that is the most funny part, you (DM) and the players are writing at that moment in real time. If the BBEG was killed because a player character had the idea and tryied successfully to paralyze him, then that was the story.

That's why I also insist (in other threads) to also contemplate the failure (instead the auto-success in time by rerolling), and things like that.

The more possibilities, branches, and that anything could happen at any time, is the most funny part. The adventures texts should only be a guidelines for directing the game in an environment, not more.

And agree, separate thread.

Zilong
2018-11-29, 05:06 AM
As others have said, typically monks will play fairly similar to one another on a mechanical level. That said they are also a great vehicle for narrative description. Sure you flurry of blows often, but what does that look like. Is it a devestating hail of punches? How bout a series of whirling kicks made breakdance style?is the stunning strike an almighty staggering blow or is it a precise jab against a pressure point?

I played a one shot with a sun soul not too long ago. It was a drow devoted to Eilistraee. I described the radiant bolts as silver crescent blades that I launched from my scimitar as I danced. This probably applies to any class, but a lot of the fun will be in the flavor.

On the tangent of fudging: it’s funny that any time it comes up people are accused of having bad-wrong fun. There are vanishingly few times when the word “never” is true. At my tables I tried letting the dice dictate everything. My players hated it. For us fudging, discreetly, is the way I create the most fun for my players. You can argue the point all you want, but experience tells me that “never” is not accurate when it comes to fudging.

LudicSavant
2018-11-29, 05:15 AM
Always?
Speed is power. Sadly not many people really grasp it around here

I agree! Grasping this makes a huge difference. :smallbiggrin:


I'm genuinely interested in what you'll throw out here.
Disclaimer: don't know for Malifice, but for me, any magic item is out. Any concentration spell is out. Any effect that lasts less than 8 hour is out.
I guess everything else is fair game.
Only thing I see right now is really the Steed spells.

Sure! Malifice seems to have disappeared after being asked to nail down his goalpost, so might as well talk to someone who will instead. :smalltongue:

A few examples comparing to the 60 foot move speed at level 18 (which is when the Monks get it) that meet your listed criteria:

The Find Steed spells of course have already been mentioned, which covers Paladins and Bards. They can also throw in things like a Swords Bard's Master's Flourish, or a Vengeance Paladin's reaction move (which can open up some mobility tricks that Monks just plain can't do).

A Storm Sorcerer, of course, grabs a 60-foot flying speed at the same level a Monk gets a 60 foot walking speed that can go up walls. Bunches of others are also getting sub-60 ongoing flying speeds at this point, which depending on your persuasion you might find better than a somewhat faster land speed.

A Moon Druid would also fit said criteria, as they just sort of live in elemental form by that point (each wildshape lasts 9 hours), and that can be an air elemental (90 feet hover speed, Air Form, and immunity to a big fat list of CC effects) or an Earth Elemental (sure, it only moves 30 feet, but burrowing and Earth Glide are rare and precious movement styles that opens up all kinds of powerful tactics).

Even the Transmuter Wizard comes pretty close to the Monk's base speed at 50 feet (stone + Longstrider mastery, which doesn't use a spell slot or Concentration) and adds +10 move speed to everyone else in the party all day while they're at it. They're also contributing rituals like Phantom Steed, which (due to the potential to cast rituals while moving) can be maintained all day for the whole party, and allows 200 foot movement speeds. Of course, said steeds are fragile, but nonetheless provide excellent openers (as well as just meaning that some monsters can't participate in the encounter at all, similar to the issue with flying folks). And of course they can do things that aren't just pure movement speed, like excavating walls with cantrips or making everyone breathe underwater all day with a single Water Breathing ritual, opening up entire different areas of the map.

So that's Wizards, Sorcerers, Druids, Paladins, and Bards that are all somewhere up there... and that's of course before they use their best mobility tools, which we're leaving out here.

Edit:

PS: Another thing that's worth noting about Monks is that in practical terms, Monk mobility often needs to be more than other characters to accomplish similar things due to their focus on melee, which means they need to get past more obstacles, and to weave in further before weaving out (thus eating up more movement). I mean, yeah, they can have a bow (or be a proud saiyan warrior), but they only get so much out of it compared to others.